[go: up one dir, main page]

0% found this document useful (0 votes)
56 views97 pages

Notes

The document outlines the classical pathway of the complement system, detailing the activation of C1 through to the formation of the membrane attack complex (MAC) and its functions. It also includes a weekly study plan for reading and reviewing materials, along with various medical case scenarios and their appropriate management strategies. Key topics covered include immunoglobulin administration for measles exposure, ethical considerations in patient treatment, and diagnostic approaches for various medical conditions.

Uploaded by

Shankar Kumar
Copyright
© © All Rights Reserved
We take content rights seriously. If you suspect this is your content, claim it here.
Available Formats
Download as DOCX, PDF, TXT or read online on Scribd
0% found this document useful (0 votes)
56 views97 pages

Notes

The document outlines the classical pathway of the complement system, detailing the activation of C1 through to the formation of the membrane attack complex (MAC) and its functions. It also includes a weekly study plan for reading and reviewing materials, along with various medical case scenarios and their appropriate management strategies. Key topics covered include immunoglobulin administration for measles exposure, ethical considerations in patient treatment, and diagnostic approaches for various medical conditions.

Uploaded by

Shankar Kumar
Copyright
© © All Rights Reserved
We take content rights seriously. If you suspect this is your content, claim it here.
Available Formats
Download as DOCX, PDF, TXT or read online on Scribd
You are on page 1/ 97

Summary of Classical Pathway:

Step Event
1️⃣ Antigen-antibody complex activates C1 (C1q, C1r, C1s)
2️⃣ C1s cleaves C4 and C2 → forms C4b2a (C3 convertase)
3️⃣ C3 convertase cleaves C3 → C3b (opsonization) and C3a (inflammation)
4️⃣ C3b joins C3 convertase → C5 convertase (C4b2a3b)
5️⃣ C5 is cleaved → C5a (inflammation) & C5b (starts MAC)
6️⃣ MAC (C5b6789) forms → pathogen lysis

🔥 Functions of Key Components:


Component Function
C3a, C5a Inflammation & chemotaxis (anaphylatoxins)
C3b Opsonization (tags microbes)
C5b-9 (MAC) Pokes holes in pathogen → cell death

Weekly Plan:
Week Focus Details
Week 1 (Day 1–7) 📘 Book Read 25 pages/day = 175 pages
Week 2 (Day 8–
📘 Book Read 25 pages/day = 175 pages
14)
Week 3 (Day 15–
📘 Book Read 25 pages/day = 175 pages
21)
Week 4 (Day 22–
📘 Book Finish last 75 pages
24)
📝 Past Papers + Do 25 pages/day past papers = 150 pages + review
Day 25–30
Review weak areas

🧠 Smart Daily Structure


Time Task
Read 15 pages of
Morning (fresh mind)
the book
Read 10 pages +
Afternoon or Evening
revise
1 past paper section
Late Evening / Night
OR review a topic
MOCK TEST

38. An 8 months' child with history of exposure to measles. Which of the following is the
most appropriate management?

Answer: B. Give immunoglobulin

Explanation: For an 8-month-old child exposed to measles, passive immunization with


immunoglobulin is the most appropriate management. This provides immediate, temporary
protection against the virus, which is crucial for infants who are too young to have completed
their measles vaccination series (typically given around 12-15 months of age). Administer
vaccine (A) is not ideal for immediate post-exposure prophylaxis in an infant this young who
may not mount an effective immune response quickly enough. Administer antivirals (D) is not a
standard post-exposure prophylaxis for measles. Reassurance (C) alone is insufficient given the
severity of measles.

39. A 68-year-old woman with metastatic lymphoma presents to the emergency department
with fever, dysuria, and back pain. She is diagnosed with pyelonephritis and antibiotics are
prescribed. The patient, however, wants to fight through with antibiotics because it's going
to be a full moon tonight and that will certainly give her strength. Her husband arrives at the
hospital and wants her to receive antibiotics. Which of the following is the most appropriate
next step in managing this patient?

Answer: B. Assess the patient's decision-making capacity

Explanation: This is a complex ethical scenario. The patient is refusing treatment based on a
non-medical belief ("full moon tonight"). While her husband wants her to receive treatment, the
patient's autonomy is paramount if she has the capacity to make her own medical decisions.
Therefore, the first and most appropriate step is to assess her decision-making capacity. If
she has capacity, her wishes should be respected, even if they seem irrational to others. If she
lacks capacity, then the husband, as a surrogate decision-maker, would be consulted, and
treatment would likely proceed. Forcing treatment (A), discharging (D), or consulting ethics
immediately (C) without assessing capacity is inappropriate.

40. A 38 years old male with 1-week history of fever, drowsiness and right hemiparesis. On
examination, he has up-going planters and has had a seizure. LP reveals pleocytosis and
increased protein and normal glucose. What is the most likely diagnosis?

Answer: A. HSV encephalitis

Explanation: The clinical picture of fever, drowsiness, right hemiparesis, up-going planters,
seizures, and CSF findings of pleocytosis, increased protein, and normal glucose is highly
suggestive of HSV encephalitis. Herpes Simplex Virus (HSV) encephalitis often presents with
focal neurological deficits, altered mental status, and seizures, and the CSF typically shows a
lymphocytic pleocytosis with elevated protein and normal glucose. Bacterial meningitis (B)
would typically show low glucose. Tuberculous meningitis (C) would also typically show low
glucose and a more prolonged course. Fungal meningitis (D) is less common and often seen in
immunocompromised individuals, and CSF findings can vary but usually include low glucose.

43. A known diabetic hypertensive with deranged LFTs, cholesterol 230 mg/dL, triglycerides
130 mg/dL, HDL 45 mg/dL and family history of MI. Which of the following drugs should be
administered to this patient?

Answer: D. Rosuvastatin

Explanation: This patient has multiple risk factors for cardiovascular disease: diabetes,
hypertension, dyslipidemia (high total cholesterol, although HDL is decent), and a family history
of MI. They also have deranged LFTs. Rosuvastatin is a potent HMG-CoA reductase inhibitor
(statin) that effectively lowers LDL cholesterol and triglycerides, and raises HDL. While all
listed options are lipid-lowering drugs, statins are the first-line treatment for dyslipidemia,
especially in patients with diabetes and other cardiovascular risk factors. Rosuvastatin is
generally well-tolerated and effective. Niacin (A) can cause flushing and has less clear mortality
benefits. Clofibrate (B) and Simvastatin (C) are older or less potent options compared to
Rosuvastatin for this high-risk profile.

44. A 27-year-old woman at 39 weeks' gestation is in a car accident and bleeding profusely.
Fetal monitoring shows late decelerations. Emergency Caesarean delivery is recommended
to save the patient and fetus. The patient refuses. The husband arrives on the scene. He is
screaming that he wants everything possible to be done to save his wife and unborn son. He
says his wife is in a state of shock and not in a position to refuse treatment. He is 1a
practicing lawyer and says he will bring down the physician and the hospital if they don't act
now. Both patient and the unborn are conscious. Which of the following is the appropriate
action by the physician?

Answer: A

46. A patient with uncomplicated MI is on beta-blocker with a pulse rate of 54/min. What is
the most appropriate management?

Answer: A. Lower the dose of beta-blocker

Explanation: Beta-blockers are important in the management of MI, but they can cause
bradycardia. A pulse rate of 54/min is on the lower side, indicating that the beta-blocker might be
causing symptomatic bradycardia or pushing the limits of safety. In this situation, the most
appropriate management is to lower the dose of the beta-blocker to alleviate the bradycardia
while still providing some of its beneficial effects. Ordering EKG (B) might be done, but the
immediate management is dosage adjustment. Referral to cardiologist (C) or switching to ACE
inhibitors (D) are not the primary immediate responses to beta-blocker induced bradycardia.

48. A kid has scabies and tells you that his elder brother also has similar complaints. What is
the best management in this case?

Answer: C. Treat the whole family

Explanation: Scabies is a highly contagious skin infestation caused by mites. It spreads easily
through close physical contact. Therefore, when one family member is diagnosed with scabies, it
is crucial to treat the entire household (the whole family), even if other family members are
asymptomatic, to prevent re-infestation and control the spread. Treating only the affected child
(A) or the child and brother (B) will likely lead to recurrence. Treating the kid and the brother,
then members if they develop symptoms (D), is not effective enough for such a contagious
condition.

49. Secondary hyperparathyroidism is most often characterized by:

Answer: A. High serum PTH, high urinary phosphate

Explanation: Secondary hyperparathyroidism is a compensatory response to a prolonged


stimulus, most commonly chronic kidney disease leading to hypocalcemia and
hyperphosphatemia. The key features are:

 High serum PTH: The parathyroid glands increase PTH production in an attempt to raise
calcium levels and excrete phosphate.

 High urinary phosphate: PTH promotes phosphate excretion in the urine, but in CKD, this
mechanism becomes impaired.

 Low serum calcium (often initial trigger): Although the body tries to compensate, calcium can
be normal or low.

52. Uncontrolled diabetes mellitus and hypertension in a 55-year-old female. You suspect
her hypertension is secondary to Cushing's syndrome. What test will you order for this
patient?

Answer: A. Low-dose dexamethasone suppression test

Explanation: Cushing's syndrome is characterized by excessive cortisol production. The low-


dose dexamethasone suppression test is the initial screening test for Cushing's syndrome.
Dexamethasone is a potent synthetic glucocorticoid; in a healthy individual, it suppresses ACTH
release from the pituitary, leading to a decrease in cortisol. In Cushing's syndrome, cortisol
production is not suppressed. High-dose dexamethasone suppression test (B) is used to
differentiate between Cushing's disease (pituitary adenoma) and other causes of Cushing's
syndrome after the initial diagnosis. 24-hour urinary cortisol (C) is also a good initial screening
test, but the low-dose dexamethasone suppression test is a common choice. Urinary VMA levels
(D) are for pheochromocytoma.

55. What's the first thing to give in diabetic neuropathy?

Answer: A. Pregabalin

Explanation: For symptomatic relief of neuropathic pain in diabetic neuropathy, Pregabalin is a


first-line agent. Gabapentin is another similar option. Tricyclic antidepressants (TCAs) like
amitriptyline are also used, but pregabalin/gabapentin are often preferred due to better
tolerability profile. Propofol (B) is an anesthetic. Fentanyl (C) is an opioid, generally reserved
for severe pain. Tramadol (D) is a weaker opioid.

56. What's the 1st sign of compartment syndrome?

Answer: D. Paresthesia

Explanation: The classic "5 Ps" of compartment syndrome are:

1. Pain (out of proportion to injury, unrelieved by analgesics)

2. Paresthesia (numbness, tingling) - often one of the earliest neurological signs.

3. Pallor (pale skin)

4. Paralysis (loss of function)

5. Pulselessness (a late and ominous sign)

While pain is the most consistent and often first symptom, paresthesia (altered sensation,
numbness, tingling) is often one of the earliest neurological signs indicating nerve compression,
making it a very important early indicator. Tenderness and tightness are physical findings, not
subjective signs.

58. A medical student develops tachycardia. His ECG is shown. What's the diagnosis?

Answer: B. SVT

Explanation: The image of the ECG is not provided, but if a medical student develops
tachycardia and the ECG shows a Supraventricular Tachycardia (SVT), it's typically
characterized by a narrow QRS complex tachycardia with a regular rhythm. VT (A) usually
has a wide QRS. Atrial fibrillation (C) is irregularly irregular. WPW (D) is a pre-excitation
syndrome that can lead to SVT or wide complex tachycardia depending on the conduction.
Without the ECG, it's an assumption based on "SVT" being a common and usually benign
arrhythmia in young, otherwise healthy individuals under stress.

60. A pregnant lady comes with history of TB. After delivery what to do?

Answer: A. Give baby 6 months of INH

Explanation: This question concerns prophylaxis for a neonate born to a mother with a history
of TB. If the mother has active TB, the baby should be given isoniazid (INH) prophylaxis. Even
if the mother's TB is considered treated, exposure during pregnancy or birth can put the baby at
risk. Giving the baby 6 months of INH (Isoniazid) is a standard prophylactic measure to prevent
TB infection in high-risk infants. Starting anti-TB drugs only after symptoms appear (C) is
reactive and risky. Starting baby on ATT (B) implies full anti-TB treatment which is not needed
for prophylaxis. "Tell mother to have no contact with baby" (D) is generally not practical or
advisable unless the mother has highly contagious active disease and cannot be treated.

61. Most common worldwide cause of bronchiectasis?

Answer: A. TB

Explanation: While Cystic Fibrosis (B) is a common cause of bronchiectasis in developed


countries, Tuberculosis (TB) is the most common worldwide cause of bronchiectasis,
especially in endemic areas. TB can cause extensive lung damage and bronchial distortion,
leading to bronchiectasis. Stomach drainage (C) and along celiac nodes (D) are not causes of
bronchiectasis.

62. 28-year-old male with a history of chronic nasal congestion now with progressively
worsening dyspnea. He took aspirin recently for myalgias. Physical exam reveals expiratory
wheeze and a polyp in left nasal turbinate. What is the cause?

Answer: B. Aspirin associated asthma

Explanation: This constellation of symptoms (chronic nasal congestion, nasal polyp, worsening
dyspnea, expiratory wheeze, and trigger by aspirin) is highly suggestive of Aspirin-
Exacerbated Respiratory Disease (AERD), also known as aspirin-induced asthma or Samter's
Triad. This condition involves asthma, chronic rhinosinusitis with nasal polyps, and sensitivity to
aspirin and other NSAIDs. Allergic rhinitis (A) alone wouldn't explain the aspirin sensitivity and
wheeze. Bronchiectasis (C) and Upper respiratory tract infection (D) are less likely to present
with this specific triad.

63. Patient presents to the ER with a tension pneumothorax, needle aspiration is planned.
What is the appropriate level for needle aspiration?

Answer: A. 2nd intercostal space, midclavicular line


65. 16 months' baby with stridor. Increased when baby cries. Normal otherwise. What to do?

Answer: A. Reassure

Explanation: Stridor that is intermittent and increases with crying but is normal otherwise
in a 16-month-old baby (which is past the typical age for congenital laryngomalacia, usually
resolving by 12-18 months) suggests a benign, likely transient cause, or a mild form of
laryngomalacia that is resolving. If the baby is otherwise normal (no respiratory distress, good
feeding, no fever, etc.), reassurance and watchful waiting might be appropriate. Further
investigation would be needed if it progresses or causes distress. Urgent intervention (B, C, D) is
not indicated if the stridor is only exacerbated by crying and the baby is otherwise well.

68. Hyperthyroid picture: which investigation to order?

Answer: A. TSH

Explanation: The initial and most sensitive investigation for suspected hyperthyroidism is
TSH (Thyroid-Stimulating Hormone). In primary hyperthyroidism, TSH will be suppressed
(low). Once TSH is found to be low, then free T4 (B) and free T3 (C) are measured to confirm
the diagnosis and determine the severity. Antibodies (D) (e.g., TRAb, anti-TPO) are then used to
determine the cause (e.g., Grave's disease).

69. p-value of a test is set at 0.025. You will reject the null hypothesis:

Answer: C. At level of 5% significance

70. A patient present with left-sided ptosis, headache and anisocoria. He has a history of
long-standing diabetes mellitus. The patient is afebrile. Which of the following is unlikely
cause?

Answer: C. Myasthenia gravis

Explanation: The symptoms of left-sided ptosis, headache, and anisocoria (unequal pupil
size) are highly suggestive of a third cranial nerve (oculomotor nerve) palsy. In a patient with
long-standing diabetes and being afebrile, this is very classic for a diabetic (ischemic) third
nerve palsy, which is typically pupil-sparing (i.e., anisocoria is not usually prominent, or the
pupil is normal/minimally involved while other muscles are affected). However, the inclusion of
anisocoria makes one consider a compressive lesion (like an aneurysm).

 Meningitis (A) would typically present with fever and other signs of meningeal irritation, which
are absent (afebrile). Therefore, meningitis is an unlikely cause.

 Space occupying lesion (B) (e.g., aneurysm) can cause 3rd nerve palsy with anisocoria.
 Diabetic neuropathy (D) is the most common cause of isolated cranial neuropathies in
diabetics.

Myasthenia gravis (C) can cause ptosis and ophthalmoplegia (affecting eye movements) and
sometimes anisocoria, but it is typically fluctuating and fatigable, and a headache would be less
typical for its primary presentation. However, given the context of a "third nerve palsy"
presentation, myasthenia gravis is not the classic first thought.

Re-evaluation: If the patient has ptosis, headache, and anisocoria, it is a 3rd nerve palsy.

 Meningitis: unlikely because afebrile.

 Space occupying lesion: possible, especially with anisocoria.

 Diabetic neuropathy: possible, but ischemic 3rd nerve palsy is usually pupil-sparing. If the
anisocoria is significant, it points away from simple diabetic neuropathy.

 Myasthenia Gravis: can cause ptosis and ophthalmoplegia but usually fluctuating, no headache.

The question asks for the unlikely cause. Meningitis, given the "afebrile" status, is quite
unlikely. Myasthenia gravis, while causing ptosis, usually lacks headache and isn't a typical
isolated 3rd nerve palsy presentation with anisocoria as a prominent feature. Both could be
considered unlikely depending on the precise nature of the anisocoria and headache. However,
meningitis is very strongly ruled out by the lack of fever in an acute presentation.

71. A case history of mother with history of Sheehan syndrome. All labs show
hypopituitarism. The lesion is most likely at which site?

Answer: B. Pituitary gland

Explanation: Sheehan's syndrome is postpartum pituitary necrosis caused by ischemic damage


to the pituitary gland following severe peripartum hemorrhage or shock. It leads to varying
degrees of hypopituitarism (deficiency of pituitary hormones). Therefore, the lesion is most
likely at the Pituitary gland.

72. Obese man presents with hypertension: the most relevant dietary intervention to reduce
blood pressure would be:

Answer:

Correct answer for dietary intervention to reduce BP in an obese man is a low sodium diet
and weight loss through caloric restriction. Given options, 'Decrease sodium' is the best fit.
73. A drowsy 11 months old baby with sunken fontanelle: What is the most appro...

Answer: D. IVF hydration and observe (assuming the option says "IV hydration and observe")

Explanation: A drowsy 11-month-old baby with a sunken fontanelle indicates dehydration.


The most appropriate initial management for significant dehydration in a drowsy infant is rapid
intravenous (IV) fluid resuscitation/hydration followed by observation and further
management. Oral rehydration (A) might be tried if the child is not drowsy. Referrals (B, C) are
not the immediate management.

76. A child with diarrhea has 10 RBCs and many WBCs on stool D/R. What is the most likely
cause?

Answer: B. Shigella

Explanation: Stool containing RBCs (red blood cells) and many WBCs (white blood cells)
indicates an inflammatory or invasive diarrhea. Among the given options, Shigella is a classic
cause of dysentery, characterized by invasive infection leading to bloody (RBCs) and
inflammatory (WBCs) diarrhea. Rotavirus (D) causes watery diarrhea, typically without blood or
significant WBCs. Salmonella (C) can cause invasive diarrhea, but Shigella is a very strong
match for this presentation. Entamoeba histolytica (B) also causes amoebic dysentery with blood
and WBCs, but Shigella is a more common bacterial cause fitting this description.

77. 14-year-old patient presented with Low-fever, anorexia, multiple sub-cm LN in posterior
triangle, tender, smooth liver edge 2 cm BCM, diagnosis?

Answer: B. Infectious mononucleosis

Explanation: The presentation of a 14-year-old with low-grade fever, anorexia, multiple


tender lymph nodes (especially in the posterior triangle), and a tender, smooth
hepatomegaly (liver edge 2 cm below costal margin) is highly characteristic of Infectious
Mononucleosis, caused by the Epstein-Barr virus (EBV). Liver abscess (A) would typically
present with higher fever and more localized pain. Lymphoma (C) can cause lymphadenopathy
and hepatomegaly, but the "tender" nodes and the overall picture in a 14-year-old make
infectious mononucleosis more likely as a first thought. Hepatitis (D) can cause hepatomegaly
and anorexia but less typically prominent lymphadenopathy.

78. Female with fever, sore throat, fatigue, spleen tip, petechiae on palate, inflamed pharynx,
after CBC, investigation you will do?

Answer: A

Explanation: The symptoms of fever, sore throat, fatigue, splenomegaly (spleen tip),
petechiae on palate, and inflamed pharynx are highly suggestive of Infectious
Mononucleosis (as in Q77). After a CBC (which would likely show atypical lymphocytosis), the
next best investigation to confirm this diagnosis is a Monospot test (heterophile antibody test).
Blood culture (B) would be done if bacterial infection is suspected (e.g., strep throat, but
petechiae on palate is also common in mono). Throat swab (C) is for strep. Bone marrow culture
(D) is not indicated.

79. Which of the following anti-tuberculous drug is contraindicated in pregnancy?

Answer: E. Streptomycin

Explanation: Streptomycin is an aminoglycoside antibiotic used in TB treatment, but it is


contraindicated in pregnancy due to its known ototoxic effects on the fetus, leading to
congenital deafness. The other first-line anti-TB drugs (Rifampicin, Pyrazinamide, Ethambutol,
Isoniazid) are generally considered safe for use in pregnancy when needed.

80. Man presents with complain of ammonia in his eye. Conjunctival injection and hazy
cornea. What to do?

Answer: B. Irrigation

Explanation: Ammonia is an alkaline substance. Alkali burns to the eye are medical
emergencies as they can cause severe and rapidly progressing damage. The immediate and most
crucial intervention is copious and prolonged irrigation of the eye to dilute and remove the
chemical. This should be done immediately, even before transport to a hospital. Antibiotics (A),
analgesia (C), or topical steroids (D) may be used later, but irrigation is the first and most critical
step.

81. Girl with fever for 3 days, on panadol and amoxicillin. Fever subsided, now rash. What is
the diagnosis

Answer: C. Roseola infantum

Explanation: As discussed previously for the incomplete question 81, a fever that subsides,
followed by the appearance of a rash in a child, is the classic presentation of Roseola infantum
(Exanthem Subitum). German measles (Rubella) (A) typically presents with a rash concurrent
with or just after fever. Measles (B) has a characteristic rash following prodromal symptoms,
often with fever persisting. Drug allergy (D) could cause a rash after medication, but the "fever
subsiding then rash" pattern is very specific for Roseola.

83. 28-year-old man with low backache for the last two years. The pain is non woke and has
become worse with time. There is stiffness in the back especially easy in the morning and it
gets better with activity. Examination is unremarkable except for no forward flexion.
Diagnosis?

Answer: A. Ankylosing spondylitis


Explanation: The key features pointing to Ankylosing Spondylitis are:

 Low backache for two years: Chronic inflammatory pain.

 Pain worse with rest (non-woke, but worse with time, implying chronic inflammation)

 Morning stiffness

 Pain improves with activity

 Reduced forward flexion of the spine.

These are classic symptoms of inflammatory back pain, which is characteristic of Ankylosing
Spondylitis, a seronegative spondyloarthropathy. Lumbar disc prolapse (B) would typically
worsen with activity or specific movements, and often cause radicular pain. Lumbosacral
arthritis (C) is too general. Rheumatoid arthritis (D) primarily affects peripheral joints, not
typically the spine as the initial presentation, and it's less common to present this way.

84. Person on thiazide diuretic present with swollen big toe and painful. What to give?

Answer: C. NSAIDs

Explanation: A patient on a thiazide diuretic presenting with a swollen, painful big toe is a
classic presentation of an acute gout attack. Thiazide diuretics are a known common precipitant
of gout due to their effect on uric acid excretion. The most appropriate initial treatment for an
acute gout attack (unless contraindicated) is an NSAID (Non-Steroidal Anti-Inflammatory
Drug).

 Allopurinol (A) is for chronic gout management (lowering uric acid), not acute attacks.

 Probenecid (B) is also for chronic gout.

 Pregabalin (D) is for neuropathic pain.

86. A 15-year-old male presents to ER with sudden onset of epigastric pain radiating to his
back along with nausea and vomiting. On examination, there is epigastric tenderness with
bruise along the umbilicus and the flanks. What is the most probable diagnosis?

Answer: A. Acute Pancreatitis

Explanation: The classic presentation of Acute Pancreatitis includes:

 Sudden onset epigastric pain radiating to the back.


 Nausea and vomiting.

 Epigastric tenderness.

 The presence of bruising around the umbilicus (Cullen's sign) and flanks (Grey Turner's
sign) are severe, though less common, signs of hemorrhagic pancreatitis, indicating
retroperitoneal bleeding, which strongly points to acute pancreatitis.

Acute cholecystitis (B) and appendicitis (C) pain patterns are different and don't typically radiate
to the back with bruising like this. Penetrating ulcer (D) can cause epigastric pain but usually
without such bruising patterns.

87. 20-week pregnant lady. Tetanus in pregnancy when to give?

Answer: D. 2 doses at least 2 weeks before delivery

Explanation: Tetanus toxoid (TT) vaccination in pregnancy is crucial for preventing neonatal
tetanus. The standard recommendation for previously unvaccinated pregnant women (or those
with unknown status) is:

 Two doses of tetanus toxoid (TT1 and TT2).

 The first dose (TT1) is given as early as possible in pregnancy, typically after the first trimester
(around 20 weeks).

 The second dose (TT2) is given at least 4 weeks after TT1, and at least two weeks before the
expected date of delivery. This ensures adequate time for the mother to develop antibodies and
transfer them to the fetus, providing passive immunity to the newborn.

Therefore, "2 doses at least 2 weeks before delivery" best captures the timing for effective
neonatal protection.

88. A patient presented to you after an accident, he is unconscious and there is blood coming
out from his right ear. What will be your first step regarding his management?

Answer: D. Secure the airway

Explanation: In any unconscious patient, especially after trauma (accident), the absolute
priority is always to secure the airway (A in the options given in the image) and ensure
adequate breathing and circulation (ABCs). Blood coming from the ear might indicate a skull
base fracture, but airway management precedes all other steps in an emergency.

 Do the CT scan (A/B) and checking BSR (B/C) are secondary investigations.
 Fast scan (C/D) refers to focused assessment with sonography for trauma, also secondary.

Re-evaluation for options provided in the image:

A. Do the CT scan

B. Check his BSR

C. FAST scan

D. Secure the airway

The priority is D. Secure the airway.

89. Vaccination in 3-month-old baby in whom no previous vaccination was given?

Answer: C. BCG, DPT, OPV, Pentavalent

Explanation: For a 3-month-old baby with no previous vaccinations, catch-up vaccination is


necessary. Standard vaccinations given around this age or due for the first dose if missed earlier
include:

 BCG (Bacille Calmette-Guérin): Given at birth or earliest contact.

 DPT (Diphtheria, Pertussis, Tetanus): First dose at 6 weeks, subsequent doses.

 OPV (Oral Polio Vaccine): First dose at birth, subsequent doses.

 Pentavalent vaccine: Contains DPT, Hepatitis B, and Hib (Haemophilus influenzae type b).
This would cover many of the necessary vaccines.

Therefore, "BCG, DPT, OPV, Pentavalent" covers the most appropriate set of vaccines for a 3-
month-old with no prior history.

90. A person travelled from Chicago to Karachi and now having diarrhea. What will the
organism for causing diarrhea in this patient?

Answer: A. E. Coli

Explanation: A person traveling from a developed country (Chicago) to a developing country


(Karachi) and developing diarrhea is a classic case of Traveler's Diarrhea. The most common
cause of Traveler's Diarrhea is Enterotoxigenic Escherichia coli (ETEC).

 E. Coli (A) is the most likely.


 Influenza (B) causes respiratory illness, not diarrhea in this context.

 Giardiasis (C) can cause traveler's diarrhea but is less common acutely than ETEC.

 Amebiasis (D) can cause dysentery but is also less common than ETEC for general traveler's
diarrhea.

92. A patient is presented to you after severe diarrhea, now he is complaining of lethargy,
drowsiness with BP=90/60, his JVP is decreased.

Answer: A. JVP ↓, SVR ↑, CO ↓

Explanation: This patient is experiencing hypovolemic shock due to severe diarrhea.

 BP=90/60 indicates hypotension.

 Lethargy and drowsiness are signs of reduced cerebral perfusion.

 Decreased JVP (Jugular Venous Pressure) is a classic sign of hypovolemia (low circulating
blood volume).

In hypovolemic shock, the body's compensatory mechanisms kick in:

 Cardiac Output (CO): Decreased due to reduced preload (low blood volume).

 Systemic Vascular Resistance (SVR): Increased due to vasoconstriction in an attempt to


maintain blood pressure and vital organ perfusion.

Therefore, the most consistent findings are JVP ↓, SVR ↑, CO ↓.

93. A 73-year-old woman presents to the emergency room complaining of severe epigastric
pain radiating to her back, nausea, and vomiting. CT scan of the abdomen demonstrates
inflammation and edema of the pancreas. A right upper quadrant ultrasound demonstrates
the presence of gallstones in the gallbladder. Which of the following is an important
prognostic sign in acute pancreatitis according to Ranson's cr1iteria?

Answer: A. Amylase level

Explanation: The patient has acute pancreatitis with gallstones (biliary pancreatitis). Ranson's
criteria are a set of prognostic indicators used to assess the severity of acute pancreatitis.

At admission, Ranson's criteria include:

 Age > 55 years


 WBC count > 16,000/microL

 Blood glucose > 200 mg/dL

 LDH > 350 IU/L

 AST > 250 IU/L

Within 48 hours, additional criteria include:

 Hct decrease > 10%

 BUN increase > 5 mg/dL

 Calcium < 8 mg/dL

 Base deficit > 4 mEq/L

 Fluid sequestration > 6 L

 Oxygen partial pressure (PaO2) < 60 mmHg

Among the given options:

 Amylase level (A): While high amylase confirms pancreatitis, its level is generally not an
important prognostic sign in Ranson's criteria. It's diagnostic, not prognostic.

 Age (B): Age > 55 years is a Ranson's criterion.

 Total bilirubin level (C): Not a Ranson's criterion.

 Lipase level (D): Like amylase, elevated lipase confirms pancreatitis but is not a prognostic
indicator in Ranson's criteria.

Therefore, Age is the only correct Ranson's criterion among the choices. The question asks for an
"important prognostic sign in acute pancreatitis according to Ranson's criteria".

94. A 15 yo child was brough tot mergency after RTA.On examination his right pupil was
dilated with decerebrate posture. Immediate CT scan showed a diffuse and biconvex hyp...
and the right frontal lobe of the brain. What will be the most probable diagnosis?

Answer: Epidural: biconvex, usually arterial.

 Subdural: crescent-shaped, usually venous, diffuse.


95. A 60-year-old male, known case of DM and gout, BMI 34, presented with the BP 190/90.
He was already taking Amlodipine 10mg. Which of the following drugs you will add along
with lifestyle modification?

Re-evaluation: Given the patient's comorbidities (DM and gout), Losartan (A) (an ARB)
would be the most appropriate addition. It's renoprotective in DM and can reduce uric acid.
Alpha-blockers are typically not a first-line add-on to a CCB unless there are other specific
indications. This question asks for the best drug. If ARBs/ACEIs are not on the option list, then
the situation changes. Assuming 'Losartan' is option A, it is the best. If 'Add alpha blocker' is the
given correct answer, it implies other factors or a specific curriculum emphasis.
Based on standard guidelines for DM and HTN: ACEIs/ARBs are usually preferred first or
second line. Losartan is an ARB. So, A is the best.

96. A pregnant patient is having symptoms of diabetes mellitus. How will you confirm the a
cases of gestational diabetes mellitus?

Answer: A. OGTT with 100g glucose

Explanation: Gestational Diabetes Mellitus (GDM) is diagnosed by screening followed by a


confirmatory oral glucose tolerance test (OGTT).

 The 75g 2-hour OGTT is widely used (e.g., by the ADA).

 The 100g 3-hour OGTT is also used (e.g., by the O'Sullivan or Carpenter-Coustan criteria) and
is specifically designed for GDM diagnosis after an initial screening.

 RBS (B) and FBG (C) alone are not sufficient for diagnosing GDM.

 Urine proteins (D) are for pre-eclampsia.

Therefore, "OGTT with 100g glucose" is a standard method for confirming GDM.

97. A woman presents to the outpatient clinic with non-bilious vomiting. Physical
examination shows visible peristalsis and positive succussion splash. What will you d?

Answer: A. Endoscopy

Explanation: The combination of non-bilious vomiting, visible peristalsis, and a positive


succussion splash strongly suggests gastric outlet obstruction.

 Visible peristalsis indicates the stomach is trying to empty against an obstruction.

 Succussion splash is a sloshing sound heard over the stomach, indicating retained fluid and gas
in a dilated stomach.
 Non-bilious vomiting suggests the obstruction is proximal to the ampulla of Vater (before bile
enters the duodenum).

The most appropriate next step for diagnosis and potential intervention in gastric outlet
obstruction is Endoscopy (A). This allows for direct visualization, identification of the cause
(e.g., ulcer, tumor, stricture), and potentially therapeutic intervention.

 Barium meal (B) is a diagnostic imaging study but endoscopy offers direct visualization.

 IV fluid resuscitation (C) is supportive, not diagnostic.

 Antibiotics and gastric lavage (D) are not primary diagnostic steps.

98. A surgeon got a cut on his hand while performing surgery. His BP is 80/50 mmHg and
pulse is 150 bpm. What will be the estimated blood loss?

Answer: D. 40%

Explanation: This patient is showing signs of Class III hypovolemic shock:

 BP 80/50 mmHg: Significantly hypotensive.

 Pulse 150 bpm: Severe tachycardia.

 The question describes a "cut on his hand," which typically wouldn't cause this level of shock.
This implies a significant internal hemorrhage not directly related to the cut, or the cut is just a
red herring for the scenario (e.g., if the surgeon accidentally cut a major vessel in the patient and
is now hypotensive due to blood loss).

However, if we are to estimate blood loss based solely on these vital signs, Class III hemorrhage
is characterized by loss of 30-40% of blood volume.

 Class I: < 15% loss

 Class II: 15-30% loss (HR 100-120, BP normal to slightly decreased)

 Class III: 30-40% loss (HR 120-140+, BP decreased, altered mental status)

 Class IV: > 40% loss (HR > 140, BP severely decreased, obtunded)

Given the severe tachycardia (150 bpm) and significant hypotension, a 40% blood loss is a
reasonable estimate for Class III/early Class IV shock.
99. A 30-year-old newly married female presented to gynae OPD with a complaint of
yellowish foul smelling vaginal discharge that usually increased during sexual intercourse.
What is the most likely organism?

Answer: D. Trichomonas

Explanation: The classic symptoms of Trichomoniasis are:

 Yellowish, frothy, foul-smelling vaginal discharge.

 Discharge often worse after sexual intercourse (due to changes in vaginal pH and stirring of
fluid).

 Vaginal itching, redness, and discomfort may also be present.

 Chlamydia (A) often causes asymptomatic cervicitis or mild discharge.

 Syphilis (B) causes chancres initially, then rash, etc., not primarily vaginal discharge.

 Gonorrhea (C) can cause purulent discharge but less typically "foul-smelling" or "frothy."

100. A 22 years old male (weight=45kg) came to ER after extensive burns of different body
parts. Burn area is calculated about 25%. Calculate how much fluid should be given to him in
first 8 hours?

Answer: C. 4500 ml

Explanation: This question requires the use of the Parkland Formula for fluid resuscitation in
burn patients.

The Parkland Formula is:

Total fluid (mL) = 4 mL x (patient's weight in kg) x (% TBSA burned)

This total fluid is given over 24 hours, with half of it given in the first 8 hours and the remaining
half over the next 16 hours.

Given:

 Weight = 45 kg

 TBSA burned = 25%

1. Calculate total fluid for 24 hours:

Total fluid = 4 mL * 45 kg * 25 = 4 * 45 * 25 = 4500 mL


2. Calculate fluid for the first 8 hours:

Fluid in first 8 hours = Total fluid / 2 = 4500 mL / 2 = 2250 mL

Re-evaluation: There's a discrepancy between my calculated answer (2250 mL) and the options
provided. Let's look at the options:

A. 9000 ml

B. 4500 ml

C. 2250 ml

D. 20000 ml

B. 4500 ml

C. 2250 ml

D. 20000 ml

So, based on the calculation, C. 2250 ml is the correct answer.

Genereic 2024
1. A 28-year-old man presents to the clinic with complaints of painful urination, started two
weeks ago, swelling and pain in his right knee, and redness in both feet accompanied by
discharge. He denies any recent trauma or systemic symptoms. He denies weight loss. His
past medical history is unremarkable, but he reports unprotected sexual intercourse one
month ago. What is the most probable diagnosis?

Answer: C

2. A 44-year-old woman presents to the emergency department with progressive weakness


and difficulty swallowing that started begin 12 hours after attending a big gathering where
she ate home-canned vegetables. On examination, she has ptosis, facial weakness, and
sluggish pupillary reactions. Muscle strength is reduced in both upper and lower limbs, and
deep tendon reflexes are absent. There are no sensory deficits. Which of the following is the
most likely diagnosis?

Answer: D. Botulism

Explanation: This is a classic presentation of Botulism. The key features are:

 Progressive weakness
 Difficulty swallowing (dysphagia)
 Ptosis, facial weakness, sluggish pupillary reactions (cranial nerve palsies)
 Reduced muscle strength in limbs
 Absent deep tendon reflexes
 No sensory deficits (purely motor paralysis)
 History of eating home-canned vegetables (a common source of Clostridium botulinum
toxin).
 The onset within 12 hours is also consistent.
 Guillain-Barré Syndrome (GBS) (A) is an ascending paralysis, but sensory deficits are
usually present, and cranial nerve involvement may be later.
 Myasthenia Gravis (C) causes fluctuating weakness and often affects ocular and bulbar
muscles, but it's typically not acute onset after a meal.
 Amyotrophic Lateral Sclerosis (ALS) (B) is a chronic, progressive neurodegenerative
disease, not an acute presentation.

3. A 28-year-old woman presents to the neurology clinic with blurry vision in her righ eye
and difficulty walking for the past two weeks. She describes eye pain worse b movement and
states that she had a similar episode of numbness in her legs si months ago, which resolved
on its own. Over the past two days, she has a noticed fatigue and difficulty maintaining her
balance. On examination, she is decreased visual acuity in the right eye, a relative afferent
pupillary defect (RAP), hypperreflexia, and increased muscle tone in the lower limbs. MRI of
the brain now multiple periventricular white matter lesions. Which of the following is the
mos probable diagnosis?

Answer: C. Multiple Sclerosis (MS)

Explanation: This is a very classic presentation for Multiple Sclerosis (MS):

 Blurry vision and eye pain with movement: Suggestive of optic neuritis, a common
initial manifestation of MS.
 Difficulty walking, fatigue, difficulty maintaining balance: Consistent with motor,
cerebellar, and fatigue symptoms of MS.
 Relapsing-remitting course: "Similar episode of numbness in her legs six months ago,
which resolved on its own." This signifies the hallmark relapsing-remitting pattern.
 Decreased visual acuity, relative afferent pupillary defect (RAPD), hyperreflexia,
increased muscle tone (spasticity): Objective neurological signs consistent with
demyelination in the CNS.
 MRI with multiple periventricular white matter lesions: The definitive diagnostic
imaging finding for MS (demyelinating plaques).
 Guillain-Barré Syndrome (A) is an acute demyelinating polyneuropathy (PNS), not CNS,
and typically presents with ascending weakness.
 Myasthenia Gravis (B) causes fluctuating muscle weakness, often ocular and bulbar, but
not sensory changes or optic neuritis.
 Neuromyelitis Optica (NMO) (C in the image, assuming E is correct for MS): NMO can
cause optic neuritis and myelitis, but the specific relapsing-remitting course and
characteristic periventricular lesions on MRI point more strongly to MS.
 Acute disseminated encephalomyelitis (ADEM) (E): ADEM is monophasic, often post-
infectious, and typically more acute and widespread.

4. A 28-year-old man presents to the emergency department with progressive weaknes in


his legs for the past three days, which has now started to affect his arms. Upon examination,
he has decreased muscle strength from a seated position, muscle weakness in the upper and
lower limbs, and decreased vibratory sensation in the feet. His cranial nerves are intact. His
vitals are stable, but he has a mild tachycardia. Which of the following the most likely
diagnosis?

Answer: A. Guillain-Barré Syndrome (GBS)

Explanation: This is a classic description of Guillain-Barré Syndrome (GBS):

 Progressive weakness in legs, now affecting arms (ascending paralysis): The hallmark
of GBS.
 Decreased muscle strength from a seated position (proximal weakness): Consistent
with GBS.
 Decreased vibratory sensation in the feet: GBS is a demyelinating polyneuropathy,
affecting sensory as well as motor nerves.
 Cranial nerves intact: While GBS can involve cranial nerves (e.g., Miller-Fisher
variant), the initial presentation often spares them.
 Mild tachycardia: Autonomic dysfunction is common in GBS.
 Amyotrophic Lateral Sclerosis (B) is a chronic motor neuron disease, not acute
progressive weakness.
 Multiple Sclerosis (C) is a CNS demyelinating disease, with different presentation (e.g.,
optic neuritis, relapsing-remitting course).
 Botulism (D) causes descending paralysis, often with cranial nerve involvement early
(e.g., ptosis, dysphagia), and is purely motor with intact sensation.

5. A mother brings her 2-year-old child to the clinic with a history of 4-6 loose stools per day
for the past 4 days. The child appears distressed, starts crying, and has tears, rolling down
their cheeks. The child is alert but irritable, and there are no sunken eyes, lethargy, or poor
skin turgor. According to the Integrated Management of Neonatal and Childhood Illnesses
(IMNCI) guidelines, what is the most appropriate initial step in managing this child's
condition?

Answer: A. Oral rehydration

Explanation: This child presents with signs of some dehydration, as per IMNCI guidelines
(distressed, irritable, tears, but no sunken eyes, lethargy, or poor skin turgor, which would
indicate severe dehydration).
 For some dehydration, the IMNCI guideline recommends oral rehydration therapy
(ORT) using Oral Rehydration Salts (ORS).
 IV fluids (B) are reserved for severe dehydration or inability to tolerate ORT.
 Stool culture (C) and antibiotics (D) are typically not the immediate first step for
mild/moderate dehydration unless there are specific signs of dysentery or severe bacterial
infection.

6. A 25-year-old female presents to the gynecology clinic with complaints of frothy, yellow-
green vaginal discharge and intense vaginal itching. She also reports dysuria and discomfort
during intercourse. An examination there is erythema and inflammation of the vaginal walls,
and the cervix appears friable with punctate bleeding spots. A wet mount microscopy of the
vaginal discharge reveals motile, flagellated protozoa. What is the most appropriate
treatment for this condition?

Answer: A. Metronidazole

Explanation: The clinical picture (frothy, yellow-green vaginal discharge, intense itching,
dysuria, discomfort with intercourse, vaginal/cervical inflammation, and motile, flagellated
protozoa on wet mount) is a classic description of Trichomoniasis. The most appropriate
treatment for Trichomoniasis is Metronidazole.

 Fluconazole (B) is for fungal infections (candidiasis).


 Doxycycline (C) is for chlamydia or gonorrhea.
 Clindamycin (D) is for bacterial vaginosis.
 Azithromycin (E) is for chlamydia or gonorrhea.

7. A patient with hyperkalemia is scheduled for surgery. The anesthesiologist is considering


which neuromuscular blocker to use and is concerned about the potential for worsening the
patient's condition. Which neuromuscular blocker is contraindicated in a patient with
hyperkalemia?

Answer: D. Succinylcholine

Explanation: Succinylcholine is a depolarizing neuromuscular blocker. It causes a transient


efflux of potassium from the intracellular to the extracellular space, which can lead to a
significant increase in serum potassium levels. In a patient with pre-existing hyperkalemia,
this can cause life-threatening arrhythmias, including cardiac arrest. Therefore, Succinylcholine
is absolutely contraindicated in patients with hyperkalemia.

 Atracurium (A), Pancuronium (B), Vecuronium (E), and Rocuronium (C) are non-
depolarizing neuromuscular blockers, which do not cause a significant potassium release
and are generally safe to use in patients with hyperkalemia.
8. A 38-year-old male is undergoing a minor skin procedure under local anesthesia with
lidocaine. Shortly after the injection, he reports a strange tingling sensation around his
mouth and a metallic taste. He also feels dizzy and lightheaded. His vitals alert. On further
questioning, he denies any nausea, vomiting, seizures, tremor or visual disturbances. The
doctor immediately suspects lidocaine toxicity. Which of the following is the most common
initial sign of lidocaine toxicity?

Answer: D

Explanation: Lidocaine toxicity typically presents with central nervous system (CNS) and
cardiovascular effects. The earliest CNS signs are usually excitatory, but can progress to
depression. Common early signs of lidocaine toxicity include:

 Circumoral numbness/perioral tingling


 Metallic taste
 Dizziness, lightheadedness
 Tinnitus (ringing in the ears)
 Visual disturbances (blurred vision)
 Slurred speech
 Nausea and vomiting

9. high doses of citalopram and escitalopram (both SSRIs) and amitriptyline (a TCA) have
been linked to prolonged Q-T intervals on ECGs, a risk factor for potentially fatal
arrhythmias.
19. A 22-year-old patient with Type I Diabetes is found unresponsive on the office floor by
coworkers. The patient has no visible injuries, and there is no history of trauma. Coworkers
report the patient last ate lunch 4 hours ago. Which initial test should the emergency team
prioritize to give immediate management?

Answer: B. Blood glucose test (BSR)

Explanation: In an unresponsive diabetic patient, the most immediate life-threatening condition


to rule out is hypoglycemia. While other conditions like stroke or diabetic ketoacidosis (DKA)
are possible, a rapid bedside blood glucose test (BSR - Blood Sugar Random or RBS - Random
Blood Sugar) is quick, non-invasive, and provides crucial information to guide immediate life-
saving treatment (e.g., IV dextrose for hypoglycemia).

 Arterial blood gas (A) and Serum electrolytes (D) are important but take longer and are not the
absolute first priority for immediate management of an unresponsive diabetic.

 Urinary ketones (C) would be part of DKA workup but not the initial test for an unresponsive
patient.
20. A 45-year-old man with myasthenia gravis is scheduled for elective surgery. The
anesthesiologist must select a neuromuscular blocking agent (NMBA) for intubation. Which
agent is most likely to exhibit resistance due to the patient's underlying condition?

Answer: A

Explanation: Patients with Myasthenia Gravis have a reduced number of functional acetylcholine
receptors at the neuromuscular junction. This makes them highly sensitive to non-depolarizing
neuromuscular blocking agents (NMBAs) like Vecuronium, Rocuronium, Pancuronium, and
Atracurium. They will often require significantly reduced doses of these agents and may exhibit
a prolonged duration of action, making it appear as "resistance" to reversal or requiring very little
to achieve blockade.

In contrast, they show resistance to depolarizing agents like succinylcholine (although


succinylcholine is generally avoided due to potential for prolonged block and hyperkalemia).
The question asks which agent is most likely to exhibit resistance, which could be interpreted as
requiring a higher dose for effect, or as being difficult to reverse. However, given myasthenia
gravis and NMBAs, the sensitivity is key.

21. A 35-year-old woman presents to her primary care physician with a 3-month history of
fatigue, generalized weakness, and pallor. She reports heavy menstrual bleeding. For the
past year, requires her to change sanitary products every 2 hours during the day. She denies
other sources of bleeding, recent weight loss, or gastrointestinal symptoms. Physical
examination reveals pale conjunctiva and brittle nails. There is no hepatosplenomegaly,
lymphadenopathy, or signs of infection. Laboratory findings: Hemoglobin: 8.5 g/dL (normal:
12-16 g/dL), MCV: 70 fL (normal: 80-100 fL), Serum ferritin: 10 ng/mL (normal: 20-200
ng/mL). What is the most appropriate initial treatment for this patient?

Answer: D. Oral ferrous sulfate supplementation

Explanation: This patient has classic signs and lab findings of Iron Deficiency Anemia (IDA):

 Fatigue, generalized weakness, pallor, brittle nails: Common symptoms.

 Heavy menstrual bleeding (menorrhagia): A very common cause of IDA in women.

 Hemoglobin 8.5 g/dL: Anemia.

 MCV 70 fL (microcytic): Consistent with IDA.

 Serum ferritin 10 ng/mL (low): Confirms iron deficiency (ferritin is a measure of iron stores).

The most appropriate initial treatment for IDA, once the cause is identified and addressed (if
possible), is Oral ferrous sulfate supplementation.

 Oral iron absorption is generally good.


 Parenteral iron (B) is for severe malabsorption or intolerance to oral iron.

 Desferrioxamine (C) is for iron overload.

 Packed red blood cell transfusion (E) is reserved for severe symptomatic anemia or
hemodynamic instability, which is not indicated here given her chronic symptoms.

 Subcutaneous erythropoietin injections (A) are for anemia of chronic kidney disease or certain
other anemias, not primary IDA.

22. A 50-year-old male presents to the clinic with complaints of progressive fatigue and
generalized weakness over the past several months. On examination, he appears pale.
Laboratory investigations reveal hemoglobin of 9 g/dL and a mean corpuscular volume
(MCV) of 72 fL. Which of the following is the most appropriate next investigation to confirm
the diagnosis?

Answer: A. Total iron-binding capacity (TIBC)

Explanation: This patient has microcytic anemia (Hb 9 g/dL, MCV 72 fL). The most common
cause of microcytic anemia is Iron Deficiency Anemia (IDA). To confirm IDA, you would
investigate iron studies.

 Total iron-binding capacity (TIBC) is a key component of iron studies. In IDA, TIBC is
typically elevated as the body tries to maximize iron absorption.

 Bone marrow biopsy (B) is invasive and not the first step for suspected IDA.

 Serum ferritin (C) would be the most direct indicator of iron stores (low in IDA), but TIBC is
also crucial. If both were options, ferritin would be slightly more specific as the initial, but TIBC
is a standard part of confirmation.

 Complete blood count (D) has already been done.

 Peripheral blood smear (E) helps characterize red cell morphology but doesn't confirm iron
deficiency itself.

If the goal is to confirm IDA, a full iron panel (serum iron, TIBC, ferritin, transferrin saturation)
is needed. Among the choices, TIBC is a standard next step to confirm suspected IDA.
25. A 42-year-old female with a history of bipolar I disorder presents for a follow-up
appointment. She was initiated on lithium carbonate three months ago, which has effectively
stabilized her mood. However, she now reports a 7 kg weight gain, worsening acne, and
increased nocturia. Her current serum lithium level is 0.9 mEq/L. She expresses significant
distress regarding the side effects and is hesitant to continue treatment. Which of the
following is the most appropriate next step in managing this patient?

Answer: C. Discontinue lithium and initiate monotherapy with lamotrigine

Explanation: This patient is experiencing common side effects of Lithium (weight gain,
worsening acne, nocturia) despite having a therapeutic lithium level (0.9 mEq/L). While lithium
has effectively stabilized her mood, the side effects are causing distress and treatment non-
adherence.

 Adjusting the dose of lithium or adding additional mood stabilizers: This is a complex
situation.

 Lowering the dose of lithium and add a topical acne treatment (A): Lowering lithium might
destabilize mood.

 Decrease lithium dose and add topical acne treatment (B): Similar to A.

 Discontinue lithium and initiate monotherapy with lamotrigine (C): Lamotrigine is an


alternative mood stabilizer, but monotherapy might not be enough for bipolar I, especially if
lithium was effective. Switching requires careful cross-titration to avoid relapse.

 Discontinue lithium and initiate combination therapy with valproic acid and an atypical
antipsychotic (D): This is a reasonable alternative regimen if lithium is causing intolerable side
effects. Valproic acid is a mood stabilizer, and an atypical antipsychotic can help with mood
stabilization.

 Continue lithium and add hydrochlorothiazide to manage weight gain and nocturia (E):
Hydrochlorothiazide can interact with lithium, increasing lithium levels and leading to toxicity.
It's not the first choice for weight gain or nocturia in this context.

Question 27: A 35-year-old male presents to the emergency room with a history of rat poison
ingestion. He is currently asymptomatic but is concerned about potential toxic effects. On
examination, his vital signs are stable. Which of the following investigations is most
appropriate to assess the extent of poisoning?

The provided options for question 27 are:

A. Liver Function Tests (LFTs)

B. Renal Function Tests (RFTs)


C. Electrolytes

D. Urinalysis

E. Prothrombin Time (PT)

Given these choices, and knowing that anticoagulant rodenticides are common, Prothrombin
Time (PT) is the most appropriate and specific test to assess the extent of poisoning for that
type, as it directly measures the effect on coagulation.

28. A 45-year-old woman presents for a routine check-up, during which her blood pressure
is recorded as 150/90 mmHg. She has a family history of essential hypertension, but her
laboratory results, including lipid profile, are normal. Her BMI is 24 kg/m2, and she has no
other cardiovascular risk factors or comorbidities. What is the most appropriate next step in
her management?

Answer: D. Refer to a cardiologist

Explanation: Her BP is elevated (Stage 1 Hypertension). While lifestyle modification is


important, a BP of 150/90 warrants further evaluation and potentially medication. Given she's
already presenting to a physician, the next step should involve a more specialized assessment to
confirm diagnosis and plan long-term management, especially considering her age. Referring to
a cardiologist is a reasonable step for comprehensive evaluation and management plan for
hypertension. Ambulatory Blood Pressure Monitoring (ABPM) (A) is a diagnostic tool, not a
management step. Lifestyle modification (B) is always relevant, but not the next or sole step for
this BP. Initiating antihypertensive medication (C) could be done by a GP, but a specialist
referral (D) is appropriate for a comprehensive approach. Perform a 24-hour urine analysis (E) is
a workup, not a management step.

29. A 5-year-old girl is brought to the clinic by her mother due to concerns about short
stature and low weight. The mother reports that the child has always been smaller than age
fellows since birth and has not shown significant catch-up growth. There are no reported
chronic illnesses, feeding difficulties, or developmental delays. What is the most appropriate
next step in evaluation?

Answer: A. Do Thyroid Profile

Explanation: Short stature and low weight in a child without apparent chronic illness or feeding
difficulties warrant investigation for endocrine causes. A thyroid profile is a common initial
screening test to rule out hypothyroidism, which can cause growth retardation. Getting
radiographic imaging (B) or a comprehensive metabolic panel (D) or referring to a pediatric
endocrinologist (E) are often done after initial screening tests like a thyroid profile or basic lab
work. Genetic tests (C) would be for specific syndromes if other workup is unrevealing.
30. A 45-year-old patient is scheduled for a laparoscopic cholecystectomy. During the
procedure, the surgeon carefully identifies and dissects Calot's triangle to locate and
preserve important structures. A thorough understanding of its anatomical boundaries and
contents is essential to avoid complications. Which of the following structures is typically
found within Calot's triangle?

Answer: Cystic arytery

Explanation: Calot's triangle is an anatomical landmark in the porta hepatis. Its boundaries are:

 Inferiorly: Cystic duct

 Medially: Common hepatic duct

 Superiorly: Inferior border of the liver

The most important structure typically found within Calot's triangle, and crucial for identification
during cholecystectomy, is the Cystic Artery. The Common Bile Duct (CBD) is not typically
considered to be within Calot's triangle but rather forms one of its boundaries (the common
hepatic duct, which joins the cystic duct to form the CBD, is one boundary).

32. A 24-month-old male is brought to the emergency department with a three-day history of
fever, increasing irritability, and poor feeding. This morning, his parents witnessed a
generalized tonic-clonic seizure lasting two minutes. On examination, he has a temperature
of 103°F (39.4°C), a bulging fontanelle, and nuchal rigidity. The child is lethargic but
arousable. Which of the following is the most appropriate next step in the management of
this patient?

Answer: A. Lumbar puncture

Explanation: This child presents with a high fever, seizure, and signs of meningeal irritation
(bulging fontanelle, nuchal rigidity, lethargy). These findings are highly suspicious for
meningitis (bacterial or viral).

The most appropriate next step for diagnosis and immediate management of suspected
meningitis in a child is a lumbar puncture (LP) to obtain cerebrospinal fluid (CSF) for analysis.

 Giving antipyretics (B) and antibiotics (C) may be part of management, but LP is crucial for
diagnosis before antibiotics.

 CT scan of the brain (D) may be done prior to LP if there are signs of increased intracranial
pressure, but LP is diagnostic for meningitis.

 Blood culture (E) is important but doesn't replace LP for meningeal signs.
33. A 60-year-old man presents to the clinic with acute, excruciating pain in his left first
metatarsophalangeal joint. He reports that the pain began abruptly overnight and has
progressively worsened. On physical examination, the joint is erythematous, swollen, warm
to the touch, and exquisitely tender. He is unable to bear weight on the affected foot. Which
of the following white blood cell types would you expect to be predominantly elevated in
condition?

Answer: D. Neutrophils

Explanation: The clinical picture (acute, excruciating pain in the first metatarsophalangeal joint,
abrupt onset, erythema, swelling, warmth, exquisite tenderness, inability to bear weight) is
classic for an acute attack of gout.

Acute gout is a highly inflammatory condition driven by urate crystal deposition. The
predominant white blood cell type involved in acute inflammation and, therefore, expected to be
elevated (leukocytosis) and seen in the synovial fluid during a gout attack are Neutrophils.

 Eosinophils (A) are for allergic reactions or parasitic infections.

 Basophils (B) are involved in allergic reactions.

 Lymphocytes (C) are for viral infections or chronic inflammation.

 Monocytes (E) are involved in chronic inflammation or phagocytosis but are not the predominant
cell type in acute gout.

34. A 28-year-old pregnant woman at 30 weeks of gestation presents to the antenatal clinic
for a routine checkup. She has been feeling mild headaches and occasional blurry vision for
the past few days but denies epigastric pain, swelling, or decreased fetal movements. Her
blood pressure is recorded as 148/92 mmHg on two separate occasions, four hours apart.
She has no history of hypertension before pregnancy. On examination, there is no significant
edema, and her reflexes are normal. Dipstick urinalysis reveals trace proteinuria. What is
the most appropriate next step in the evaluation of this patient?

Answer: C Perform a fetal ultrasound

Explanation: This pregnant woman at 30 weeks has new-onset hypertension (148/92 mmHg) and
trace proteinuria with headaches and blurry vision. These are signs suggestive of Pre-eclampsia.

The initial and most appropriate next step in evaluating pre-eclampsia involves assessing both
maternal and fetal well-being. While maternal labs (liver enzymes, creatinine, platelet count,
repeat proteinuria) are crucial, assessing the fetus is equally important.

Performing a fetal ultrasound (D) is critical to assess fetal growth (for fetal growth restriction, a
common complication of pre-eclampsia), amniotic fluid volume, and placental blood flow
(Doppler studies).
 Obtaining a 24-hour urine protein measurement (A) is also important to quantify proteinuria but
might not be the most appropriate next step if fetal well-being is not assessed.

 Starting antihypertensive therapy (B) might be needed but only after full assessment.

 Measuring serum electrolytes (C) is part of routine labs.

 Referring for cardiology consultation (E) is not the immediate next step.

Therefore, evaluating fetal well-being with an ultrasound is a key component of the initial
assessment for suspected pre-eclampsia.

35. A 30-week pregnant woman arrives at the emergency department with severe
hypertension, headache, visual disturbances, and upper abdominal pain. Her blood pressure
is 175/110 mmHg, and she has 3+ proteinuria on urinalysis. Fetal heart rate monitoring is
reassuring. What is the most appropriate initial management step for controlling her blood
pressure in this acute situation?

Answer: A. Administer IV labetalol

Explanation: This patient presents with severe pre-eclampsia (or pre-eclampsia with severe
features), indicated by BP >160/110 mmHg, headache, visual disturbances, upper abdominal
pain, and significant proteinuria. Immediate management focuses on controlling severe
hypertension to prevent maternal complications (e.g., stroke) and ensuring fetal well-being.

The first-line agents for acute severe hypertension in pregnancy include:

 IV Labetalol

 IV Hydralazine

 Oral Nifedipine (sustained release).

Among the options:

 Administer IV Hydralazine (A) is a direct vasodilator and a common choice for acute severe
hypertension in pregnancy.

 Administer IV Magnesium Sulfate (B) is for seizure prophylaxis/treatment in


pre-eclampsia/eclampsia, not for immediate BP control itself.

 Administer IV Labetalol (C) is also a good choice.

 Initiate oral Methyldopa, Nifedipine (D) are not suitable for acute severe hypertension that
requires immediate IV control.
 Immediate delivery of the baby (E) is the definitive treatment for pre-eclampsia/eclampsia but is
initiated after maternal stabilization and assessment of fetal maturity.

Given that IV Hydralazine is an option, it's a correct initial step. If both Labetalol and
Hydralazine were options and one had to be chosen, it might depend on local protocols or
specific contraindications, but both are valid.

36. A 13-year-old girl is brought to the clinic by her mother due to concerns about her short
stature and absence of menstrual periods. The mother reports that her daughter has always
been shorter than her age mates and has not developed any signs of puberty. There is no
history of chronic illness or her developmental milestones were normal. On physical
examination, the patient has a webbed neck, low-set ears, widely spaced nipples, and a
broad chest. A pelvic ultrasound reveals streak ovaries. What is the most likely diagnosis?

Answer: B. Turner Syndrome

Explanation: This is a classic presentation of Turner Syndrome (45,XO). The key features are:

 Short stature

 Primary amenorrhea (absence of menstrual periods)

 Absence of secondary sexual characteristics (lack of puberty)

 Characteristic physical features: Webbed neck, low-set ears, widely spaced nipples, broad
chest.

 Streak ovaries: Non-functional, rudimentary gonads, confirmed by ultrasound.

 Constitutional Growth Delay (A) would not typically present with the specific dysmorphic
features or streak ovaries.

 Polycystic Ovary Syndrome (PCOS) (C) presents with menstrual irregularities, hirsutism, and
obesity, not short stature or streak ovaries.

 Premature ovarian failure (D) means the ovaries stopped functioning early, but it doesn't explain
the dysmorphic features and short stature from birth.

 Congenital adrenal hyperplasia (E) affects adrenal hormone production, leading to virilization in
females, not these features.

Surgery 2024

1. A 72-year-old male with a history of chronic smoking and recently diagnosed pneumonia
presents to the emergency department with progressive confusion, fatigue, and mild nausea
over the past three days. His family notes that he has been drinking water excessively
despite having no significant thirst. He denies diarrhea, vomiting, or diuretic use. On
examination, he is alert but disoriented. His mucous membranes are moist, and there is no
peripheral edema. Laboratory tests show serum sodium: 122 mEq/L, serum osmolality:
260 mOsm/kg, urine osmolality: 500 mOsm/kg, urine sodium: 40 mEq/L. Serum creatinine
is normal. Which of the following is the most likely diagnosis?

Answer: B. Syndrome of Inappropriate Antidiuretic Hormone (SIADH)

Explanation: The patient presents with hyponatremia (serum sodium 122 mEq/L) and
hypoosmolality (serum osmolality 260 mOsm/kg) despite normal renal function and no signs
of dehydration or fluid overload. The high urine osmolality (500 mOsm/kg) and urine sodium
(40 mEq/L) indicate that the kidneys are appropriately concentrating urine and retaining
sodium, which is characteristic of SIADH. His excessive water intake without thirst further
supports this.

A 32-year-old male presents with recurrent painful oral and genital ulcers for the past
several months. He also reports episodes of blurry vision and eye pain, along with
occasional pain and swelling in his knees and ankles. Recently, he developed reddish,
tender nodules on his lower legs. On examination, he has multiple aphthous ulcers in
the oral cavity, genital ulcers, and erythema nodosum-like lesions on his shins. A
pathergy test is performed and is positive. What is the most likely diagnosis?

Answer: A. Behçet's Disease

Explanation: The classic triad of recurrent oral ulcers, genital ulcers, and eye lesions (blurry
vision, eye pain) in conjunction with skin lesions (erythema nodosum-like nodules) and a
positive pathergy test strongly points towards Behçet's Disease. Joint involvement is also
common.

A 58-year-old male with a 15-year history of poorly controlled type 2 diabetes mellitus
presents with burning pain, numbness, and tingling in both feet for the past six months.
He reports that the symptoms are worse at night and sometimes feel like "walking on
cotton." He also mentions occasional difficulty in sensing temperature changes and
frequent minor foot injuries that he only notices after some time. Examination reveals
bilateral decreased vibration sensation, absent ankle reflexes, and reduced pinprick
sensation in a stocking-glove pattern. What is the most likely diagnosis?

Answer: A. Diabetic Peripheral Neuropathy

Explanation: The long history of poorly controlled diabetes, "stocking-glove" sensory loss
(burning pain, numbness, tingling in feet, worse at night), decreased vibration and pinprick
sensation, absent ankle reflexes, and unnoticed foot injuries are all hallmark signs and
symptoms of diabetic peripheral neuropathy.
A 32-year-old woman with a history of two first-trimester miscarriages presents to the
emergency department with acute-onset right leg swelling and pain for the past 12
hours. She denies recent trauma, prolonged immobilization, or travel. She has no
known history of deep vein thrombosis (DVT) or clotting disorders. On examination,
her right leg is erythematous, warm, and tender, with a positive Homan's sign.
Laboratory tests reveal mild thrombocytopenia and a prolonged aPTT that does not
correct with a mixing study. What is the most likely diagnosis?

Answer: A. Antiphospholipid syndrome

Explanation: The patient's presentation of recurrent miscarriages and an acute DVT (leg
swelling, pain, warmth, erythema, positive Homan's sign) in the absence of other typical
DVT risk factors is highly suggestive of a thrombophilic disorder. The laboratory findings of
mild thrombocytopenia and a prolonged aPTT that does not correct with a mixing study are
classic for antiphospholipid syndrome, particularly due to the presence of a lupus
anticoagulant, which can prolong aPTT and cause both arterial and venous thrombosis, as
well as recurrent pregnancy loss.

Here are the answers to the questions from the second image, with short explanations:

A 56-year-old man with a history of cirrhosis due to chronic alcohol use presents to the
emergency department with fever, increasing abdominal distension, confusion, and
generalized weakness over the past few days. He denies any recent vomiting, diarrhea, or
upper gastrointestinal bleeding. On examination, he is febrile (102°F), hypotensive (90/60
mmHg), and mildly tachycardic (HR: 102/min). His abdomen is distended with shifting
dullness but without significant tenderness or peritoneal signs. Diagnostic paracentesis shows
an ascitic fluid WBC count of 350 cells/µL with 80% neutrophils, low protein content, and a
serum-ascitic albumin gradient (SAAG) of 1.3 g/dL. What is the most likely diagnosis?

Answer: A. Spontaneous Bacterial Peritonitis

Explanation: The patient's history of cirrhosis, new-onset fever, increasing abdominal


distension, and confusion, combined with ascitic fluid analysis showing an elevated WBC
count (specifically >250 PMNs/µL, here 350 with 80% neutrophils means 280 PMNs/µL)
and a SAAG >= 1.1 g/dL (here 1.3 g/dL), are diagnostic for spontaneous bacterial peritonitis
(SBP).

A 68-year-old male with a history of chronic obstructive pulmonary disease has been
receiving treatment for an acute exacerbation. After 5 days of therapy, he develops new-onset
fever, productive cough, and increased shortness of breath. His oxygen saturation has
dropped, and there are new infiltrates on the chest X-ray. What is the most likely diagnosis?

Answer: The classic signs of new fever, productive cough, increased SOB, and new
infiltrates on CXR strongly point to an infection, specifically Pneumonia. While ARDS can
be a complication of pneumonia, it's not the initial diagnosis for these symptoms. Pulmonary
embolism would present differently (acute onset dyspnea, pleuritic chest pain, possibly clear
CXR or specific findings). Heart failure exacerbation would typically show signs of fluid
overload and cardiomegaly. Emphysema progression is chronic, not acute with fever and new
infiltrates.

A 55-year-old male with a history of chronic alcohol use disorder presents to the emergency
department with severe, sudden-onset chest pain radiating to his back, accompanied by
difficulty swallowing and fever. The symptoms began immediately after an episode of
forceful vomiting following a binge drinking session. On examination, he is tachycardic (HR:
112 bpm), febrile (100.9°F), and hypotensive (BP: 80/60 mmHg). Physical examination
shows subcutaneous emphysema in the neck and chest wall, diminished breath sounds at the
left lung base, and Hamman's sign (crunching sound synchronized with the heartbeat). WBC:
18,000/µL (neutrophil predominance). Lactate: 4.2 mmol/L (elevated). Chest X-ray:
Pneumomediastinum, left-sided pleural effusion. CT chest with oral contrast: reveals
extravasation of contrast into the mediastinum. What is the most appropriate initial
management for this patient?

Answer: C. Initiate broad-spectrum antibiotics, fluid resuscitation, and urgent surgical


consultation

Explanation: This patient has a classic presentation of Boerhaave syndrome (esophageal


rupture) due to forceful vomiting, leading to mediastinitis and pleurisy. Key findings include
chest pain radiating to the back, fever, subcutaneous emphysema, pneumomediastinum,
pleural effusion, and extravasation of contrast. This is a surgical emergency. Initial
management involves immediate fluid resuscitation for hypotension, broad-spectrum
antibiotics to cover potential infection from esophageal contents, and urgent surgical
consultation for definitive repair.

A 22-year-old male is brought to the emergency department following a high-speed


motorcycle collision. He was not wearing a helmet and was thrown from the bike. On arrival,
he complains of severe right-sided chest pain, difficulty breathing, and lightheadedness. His
vital sign: Blood pressure: 85/50 mmHg, Heart rate: 130 bpm, Respiratory rate: 28
breaths/min, Oxygen saturation: 88% on room air. Physical examination reveals decreased
breath sounds on the left hemithorax.

Note: The question is cut off, but based on the provided information, we can infer the likely
next steps in management.

Implied Answer/Management: Given the trauma, hypoxemia, hypotension, tachycardia,


and decreased breath sounds on one side, this patient likely has a tension pneumothorax or
massive hemothorax. The immediate priority is airway, breathing, circulation (ABCs)
management, which would include needle decompression or chest tube insertion for
suspected pneumo/hemothorax, along with aggressive fluid resuscitation for shock.

You are a surgical resident assisting in a laparoscopic cholecystectomy when you realize that
you need an additional instrument to proceed with the procedure. You are scrubbed in,
wearing a sterile gown, gloves, and mask. The surgical field is fully prepped, and the
procedure is in progress. The instrument you need is in an adjacent storage room. What is the
most appropriate action to take?

Answer: D. Ask a colleague to bring what you need without leaving the operating room.

Explanation: As a scrubbed-in surgical team member, maintaining sterility is paramount.


Leaving the operating room would break sterility. The most appropriate action is to ask an
unscrubbed circulating nurse or another non-sterile colleague to retrieve the instrument for
you.

A 28-year-old obese female presents to the dermatology clinic with painful, recurrent boils in
her axillae and groin for the past two years. She describes tender, red nodules that
spontaneously rupture, draining foul-smelling pus. Over time, she has developed cord-like
scars and sinus tracts in the affected areas. The lesions worsen before her menstrual cycles,
and she has tried multiple courses of antibiotics with only temporary relief. She is a current
smoker and has a family history of similar lesions in her younger sister. On examination,
multiple inflamed nodules, sinus tracts, and hypertrophic scars are noted in both axillae and
the inguinal region. No systemic symptoms are present. What is the initial investigation of
choice for this patient?

Answer: U/S

Explanation: The patient's presentation of recurrent, painful, deep inflammatory nodules and
abscesses with sinus tracts and scarring in intertriginous areas (axillae, groin), worsening
before menses, and a family history is highly characteristic of Hidradenitis Suppurativa.

11. A 24-year-old woman presents to a clinic with a slow-growing, painless swelling on


the left side of her neck that has persisted for 10 weeks. She reports intermittent low-
grade fever, drenching night sweats, and unintentional weight loss (6 kg over 3 months).
She denies cough, shortness of breath, or contact with active TB cases. Physical
examination reveals a 4 cm x 3 cm firm, non-tender, matted mass in the left cervical
lymph node chain. Fluctuance in the central portion of the mass without overlying
erythema or warmth. No hepatosplenomegaly or other palpable lymphadenopathy.
What is the most appropriate initial diagnostic investigation for this patient?

Answer: C. Fine-needle aspiration cytology (FNAC) of the lymph node with acid-fast
bacilli (AFB) staining and culture

Explanation: The combination of a chronic, painless neck mass (lymphadenopathy) with


systemic B symptoms (low-grade fever, night sweats, weight loss) is concerning for
lymphoma or tuberculosis (scrofula). Given the description of a firm, matted lymph node,
FNAC with AFB staining and culture is the most appropriate initial diagnostic step to
differentiate between these possibilities, as well as other causes of chronic lymphadenopathy.
While excisional biopsy might be needed if FNAC is inconclusive, it's typically not the
initial investigation unless malignancy is strongly suspected and FNAC cannot provide
enough tissue. Complete blood count and ESR are general tests but not diagnostic. Chest X-
ray would be for pulmonary TB, not cervical lymphadenitis specifically.

Here are the answers to the questions from the third image, with short explanations:

A 28-year-old surgical resident is preparing for an emergency laparotomy on a patient with a


suspected bowel perforation. The resident recalls the importance of aseptic technique but is
unsure about the exact protocol for scrubbing, skin antisepsis, and draping. The patient is
prepped, and the resident contemplates the following steps: scrubbing, skin antisepsis, and
draping. Which sequence adheres to evidence-based surgical aseptic protocols to minimize
infection risk?

Answer: E 5-minute chlorhexidine scrub → Apply iodine from the incision outward →
Drape starting at the incision

Explanation: While specific scrub times can vary, a 5-minute chlorhexidine scrub is a
common standard. The general principle for applying antiseptic solution is from the clean
area (incision site) outward in concentric circles to avoid bringing contaminants into the
clean field. Draping should also start from the incision site, covering the sterile field
outwards.

A 45-year-old male undergoes a complicated peripheral fistula surgery. Twelve hours


postoperatively, he notices persistent bleeding from the wound site. He denies pain, fever, or
any new symptoms. His vital signs are stable, and there is no significant swelling or
hematoma developing at the surgical site. Hemoglobin: 12.8 g/dL, Platelet count:
200,000/µL, Prothrombin time (PT): 12.5 sec, Activated partial thromboplastin time (aPTT):
30 sec, International normalized ratio (INR): 1. On examination, the wound site shows active
oozing of blood, but there is no sign of infection, hematoma, or dehiscence. What is the most
likely cause of the bleeding?

Answer: D. Bleeding from a vessel

Explanation: The patient is experiencing persistent active oozing from the wound site, but
there are no signs of a large hematoma, generalized coagulopathy (normal labs for
hemoglobin, platelets, PT, aPTT, INR), or infection. This localized, persistent bleeding,
despite normal coagulation parameters, is most indicative of inadequate hemostasis at a
specific site, likely a small vessel that was not adequately ligated or cauterized during
surgery.

A 62-year-old male presents to the emergency department with sudden-onset weakness on


the right side of his body. He also reports double vision and difficulty moving his left eye.
His symptoms began two hours ago while he was having breakfast. He has a history of
hypertension and type 2 diabetes but no prior strokes. On examination, Motor power of right
upper and lower limb is 2/5 and motor power of left upper and lower limb is normal. There is
hyperreflexia on the right side while sensations are normal on both sides. There is ptosis,
lateral deviation, and inability to adduct on left eye while there are normal right eye
movements. Moreover, there is left-sided mydriasis and non-reactive to light. What is the
most likely localization of the lesion?

Answer: C. Left midbrain

Explanation: The patient presents with sudden-onset right-sided weakness


(hemiparesis/hemiplegia) with hyperreflexia, indicating an upper motor neuron lesion.
Concurrently, he has left-sided oculomotor nerve (CN III) palsy symptoms (ptosis, lateral
deviation, inability to adduct, mydriasis, non-reactive to light). The combination of
contralateral hemiparesis and ipsilateral CN III palsy is characteristic of a Weber's
Syndrome, which localizes the lesion to the left midbrain.

18. A 62-year-old male with a history of gallstones presents with progressive jaundice, clay-
colored stools, and pruritus for 3 weeks. Over the past 48 hours, he develops fever (39°C),
right upper quadrant pain, and oliguria. Laboratory findings include: Total bilirubin: 18
mg/dL (direct: 15 mg/dL), Alkaline phosphatase: 450 U/L, Serum creatinine: 3.8 mg/dL,
WBC: 18,000/µL (neutrophil predominance), and procalcitonin is elevated. Abdominal
ultrasound confirms common bile duct dilation with a 7 mm stone and sludge. What is the
most likely mechanism of renal failure in this patient?

However, looking at the options again:

A. Hepatorenal syndrome: Usually occurs in advanced liver disease, often


without infection, and is a diagnosis of exclusion. Unlikely the initial
mechanism.

B. Prerenal AKI secondary to dehydration from vomiting: While dehydration


can contribute, the severe infection and obstruction are more prominent.

C. Acute tubular necrosis (ATN) from sepsis-induced hypotension (ascending


cholangitis): This is the most likely and direct explanation for acute kidney
injury in the context of severe ascending cholangitis and systemic
inflammation.

D. Drug-induced interstitial nephritis from recent antibiotic use: Possible, but


less likely to be the primary or initial mechanism in this acute, severe
presentation compared to sepsis-induced ATN

19. A 48-year-old woman presents to the outpatient clinic with a painless thyroid nodule that
she noticed two months ago. She denies dysphagia, voice changes, or symptoms of
hyperthyroidism or hypothyroidism. She has no history of radiation exposure or a family
history of thyroid cancer. A 2.5 cm firm, non-tender nodule is palpated in the right thyroid
lobe. No cervical lymphadenopathy is noted. Her TSH is normal and there is no thyromegaly
with no microcalcifications or irregular margins on ultrasound. A fine-needle aspiration
(FNA) is performed, and the pathology report shows follicular cells arranged in
microfollicular pattern without nuclear atypia (Bethesda category III). What is the next step
in management?

Answer: B.

20. A 30-year-old male presents to the emergency department after sustaining burn
injuries due to an accidental fire at work. He has partial-thickness burns involving both
upper limbs and the upper back, covering approximately 15% of his total body surface
area (TBSA). There is no sign of inhalation injury (no soot in the mouth, no
hoarseness). Normal respiratory rate and oxygen saturation. Heart rate 102 bpm, blood
pressure 125/75 mmHg, capillary refill time <2 seconds, but blistering and erythema
over the upper limbs and back, with intact sensation in the affected areas. Urine output
is not yet measured. What is the most appropriate next step in management?

Re-evaluation: Given the options, and the general management of burns:

A. Immediate intubation and mechanical ventilation: Not indicated without


inhalation injury or respiratory distress.

B. Initiate fluid resuscitation using the Parkland formula: This is critical for
burns >15% TBSA to prevent burn shock and maintain organ perfusion.

C. Early excision and skin grafting: This is definitive surgical management,


not the immediate next step.

D. Empirical IV antibiotics to prevent infection: Antibiotics are generally not


given empirically for burns unless there's a clear sign of infection or full-
thickness burns.

E. Empirical IV antibiotics to prevent infection: Same as D.

Conclusion for Q20: The most appropriate immediate next step for a burn patient
with 15% TBSA is fluid resuscitation using the Parkland formula to prevent
hypovolemic shock. If option B is not the chosen answer, then it implies a different
interpretation of "most appropriate next step." However, based on standard burn
management, fluid resuscitation is paramount. If we have to pick from the given
options and assume some severity leading to immediate action, then B is the best. If
we have to assume that the question is trying to trick us into choosing the most
critical immediate life-saving measure without all the information, then perhaps
intubation is what they are aiming for if a high suspicion of airway involvement is
implied, but the text explicitly denies it. Therefore, B is the correct answer.
21. A 28-year-old male presents to the clinic with a midline neck swelling and a discharging
sinus at the level of the hyoid bone. He underwent a Sistrunk operation six months ago for a
thyroglossal cyst. The swelling has gradually increased in size over the past two months, and
he reports intermittent discharge from the sinus. There is no fever or systemic symptoms. On
examination, there is a 2 cm, non-tender, midline swelling. The previous surgical scar is
visible. There is a small sinus tract with clear discharge but no cervical lymphadenopathy.
What is the next best step in management?

Answer: C. Surgical exploration and excision of the recurrent tract

Explanation: The patient presents with a recurrence of a thyroglossal duct cyst,


characterized by a midline neck swelling and a discharging sinus after a previous Sistrunk
operation. This indicates incomplete removal of the tract. The most definitive treatment for a
recurrent thyroglossal duct cyst is surgical exploration and complete excision of the
remaining tract.

24. A 25-year-old woman, who is 8 weeks pregnant, presents to the clinic with complaints of
palpitations, excessive sweating, weight loss despite a good appetite, and heat intolerance.
She also reports experiencing mood swings and anxiety over the past few weeks. On
examination, her pulse is 110 bpm, and she has a fine tremor in her hands. Thyroid
examination reveals diffuse enlargement without tenderness or nodules. No ophthalmopathy
or pretibial myxedema is noted. Laboratory investigations: TSH <0.01 mIU/L (Low), Free
T4: Elevated, Total T3: Elevated. Which of the following is the drug of choice for managing
her condition at this stage of pregnancy?

Answer: B. Propylthiouracil (PTU)

Explanation: The patient presents with symptoms and lab findings consistent with
hyperthyroidism (Graves' disease) during pregnancy. Propylthiouracil (PTU) is the preferred
antithyroid drug during the first trimester of pregnancy due to a lower risk of teratogenicity
compared to methimazole. Methimazole is generally preferred in the second and third
trimesters.

25. A 62-year-old postmenopausal woman presents to the gynecology clinic with intermittent
vaginal bleeding for the past two months. She describes the bleeding as light but recurrent,
occurring between regular periods of abstinence. She denies any pelvic pain, weight loss,
fever, or recent trauma. Her last menstrual period was 10 years ago. She has a history of
hypertension, well controlled on medications, but no prior history of gynecologic
malignancies. Her vitals are BP: 130/80 mmHg, HR: 78 bpm. On pelvic exam, no obvious
cervical lesions or vaginal discharge. The uterus is non-tender, non-enlarged, and adnexa are
unremarkable. Which of the following is the investigation of choice to establish the
diagnosis?

Answer: B. Transvaginal ultrasound (TVUS)


Explanation: Any postmenopausal vaginal bleeding requires investigation to rule out
endometrial cancer. The initial investigation of choice is usually a transvaginal ultrasound
(TVUS) to assess endometrial thickness. If the endometrial thickness is abnormal, further
investigation like endometrial biopsy is performed.

28. A 55-year-old male with a history of peptic ulcer disease. The patient undergoes
endoscopy, and a duodenal wall ulcer was diagnosed. The patient is administered midazolam
for therapeutic management. Which vessel is most likely to be involved and possibly ligated
to manage this complication?

Answer: B. Gastroduodenal artery

Explanation: The gastroduodenal artery (GDA) lies posterior to the duodenal bulb and is the
most common vessel involved in bleeding from duodenal ulcers, especially posterior ulcers.
Ligation of the GDA is a critical surgical step if endoscopic hemostasis fails.

29. A 48-year-old female patient who underwent laparoscopic sleeve gastrectomy for morbid
obesity presents with symptoms of anorexia, fatigue, and anemia. Considering her surgical
history and current symptoms, which of the following deficiencies is most likely responsible
for his condition?

Answer: B. Iron

Explanation: After bariatric surgery like sleeve gastrectomy, malabsorption of several


nutrients can occur due to changes in anatomy and digestive processes. Iron deficiency
anemia is a very common complication due to reduced acid secretion (affecting iron
absorption) and reduced surface area.

30. A patient undergoes laryngoscopy and endotracheal tube (ETT) insertion. Which of the
following hemodynamic changes is most commonly observed immediately following these
procedures?

Answer: Tachycardia and hypertensiom


Explanation: The most commonly observed hemodynamic change
immediately following laryngoscopy and endotracheal tube (ETT) insertion
is increased heart rate and blood pressure. This is due to a sympathetic
nervous system response triggered by the stimulation of the airway during
laryngoscopy, leading to the release of adrenaline and noradrenaline,
which causes vasoconstriction and increased cardiac output.

33. A 32-year-old male is brought to the emergency department after a road traffic accident
(RTA). He was the driver of a vehicle involved in a high-impact collision. Upon arrival, he is
conscious but drowsy. Vital signs are heart rate: 110 bpm, Blood pressure: 100/70 mmHg,
Respiratory rate: initially 14/min, now decreased to 8/min with oxygen saturation 80% on
room air. On examination, there is severe chest pain and paradoxical chest wall movement,
multiple rib fractures and decreased breath sounds bilaterally with GCS: 9. Arterial blood gas
(ABG) results are pO2: 40 mmHg, pCO2: 60 mmHg and pH: 7.28. What is the most
appropriate next step in management?

Answer: E. Perform endotracheal intubation and mechanical ventilation

Explanation: The patient is conscious but drowsy, has paradoxical chest wall movement
(suggesting flail chest), severe respiratory distress (respiratory rate decreased from 14 to 8,
O2 saturation 80%), and evidence of respiratory failure (hypoxemia, hypercapnia, acidosis on
ABG). This indicates impending or actual respiratory arrest. Endotracheal intubation and
mechanical ventilation are urgently required to secure the airway, improve oxygenation and
ventilation, and stabilize the patient.

34. A 45-year-old male with a history of symptomatic gallstones undergoes an elective


laparoscopic cholecystectomy. During the procedure, the surgeon identifies Calot's triangle.
However, brisk bleeding is encountered while dissecting within the triangle, requiring
hemostasis. During a cholecystectomy, injury to which structure within Calot's triangle is
most common?

Answer: E Cystic Artery

Explanation: Calot's triangle is an anatomical landmark in cholecystectomy. Its boundaries


are the cystic duct, common hepatic duct, and inferior border of the liver. While the cystic
artery is often ligated within this triangle,

was involved in a motorcycle accident and sustained a closed femoral shaft fracture. He
underwent intramedullary nailing of the femur two days ago. On postoperative day 2,
the patient reports increasing pain, tightness, and swelling in the operated leg. He also
notices numbness and tingling in his foot. Examination findings include affected limb is
edematous, tense, pale, pain is exacerbated with passive movement of the toes,
pulsations are reduced over the dorsum of the foot and posterior tibial but diminished.
The surgical site appears intact, and there is no obvious hematoma or wound infection.
What is the next best step in the management of this patient?

Explanation: The patient has classic signs and symptoms of acute compartment syndrome
(increasing pain, tightness, swelling, numbness/tingling, pain with passive stretch,
diminished pulses, pallor, tense limb) following a long bone fracture and surgery. This is a
surgical emergency. The next best step is not imaging for DVT or a fasciotomy without
confirming the diagnosis. The immediate step for suspected compartment syndrome is to
measure compartment pressures. 39. A 67-year-old male with a history of diverticular
disease presents to the emergency department with severe lower abdominal pain, fever,
and vomiting for two days. His vitals are: Temperature: 102.2°F (39°C), Heart rate: 115
bpm, Blood pressure: 100/60 mmHg. On examination, abdomen is rigid, diffuse
tenderness, guarding all over abdomen and bowel sounds are absent. Laboratory
results are WBC count: 18,500/µL (elevated with neutrophilia), Lactate: 3.8 mmol/L.
Creatinine: 1.2 mg/dL and CRP levels are elevated. CT abdomen with contrast:
Perforated sigmoid diverticulitis with free air and fluid in the peritoneal cavity. The
patient is scheduled for emergency surgery. Which of the following antibiotic regimens
should be administered postoperatively?

Answer: B. Piperacillin-tazobactam

Explanation: The patient has perforated sigmoid diverticulitis with free air and fluid,
indicating peritonitis. This is a severe intra-abdominal infection requiring broad-spectrum
antibiotics to cover gram-negative enteric bacteria and anaerobes. Piperacillin-tazobactam is
an excellent choice as it provides broad-spectrum coverage against these common pathogens
in intra-abdominal infections.

40. A 38-year-old woman presents to the emergency department with sudden-onset, severe
left-sided flank pain that began an hour ago. She is G3P2. She also reports nausea and
difficulty urinating, but denies fever or hematuria. On examination, abdomen is soft but left
costovertebral angle (CVA) tenderness. Urine dipstick shows microscopic hematuria. What is
the investigation of choice to make a definitive diagnosis in this patient?

Answer: A CT abdomen/pelvis without contrast.


Explanation: The patient's symptoms (sudden-onset severe flank pain radiating to the CVA,
nausea, difficulty urinating, microscopic hematuria) are classic for renal colic due to
urolithiasis (kidney stone). While CT abdomen and pelvis without contrast is now the gold
standard for diagnosing kidney stones, Intravenous Pyelogram (IVP) (option D) was a
traditional method for visualizing the urinary tract and identifying stones, especially in older
contexts or when CT is not readily available. However, given modern practice, CT without
contrast is usually preferred due to speed and avoiding contrast in renal dysfunction.

44. A 55-year-old male is brought to the emergency department after a motor vehicle
accident. He is pale, diaphoretic, and confused. His initial vital signs are Blood pressure:
80/50 mmHg, Heart rate: 130 bpm, Respiratory rate: 24/min and Oxygen saturation: 95% on
room air. The patient is suspected to have hypovolemic shock due to intra-abdominal
bleeding. He is started on aggressive fluid resuscitation with crystalloids, and transfusion of
packed red blood cells. After 2 liters of intravenous fluid, his blood pressure improves to
100/65 mmHg, and his heart rate decreases to 100 bpm. However, he still appears lethargic.
Which of the following is the best indicator of adequate resuscitation in this patient?

Answer: C. Central venous pressure (CVP)

Explanation: While blood pressure and heart rate are important, in a patient with ongoing
shock and massive fluid resuscitation, Central Venous Pressure (CVP) is a better indicator
of adequate intravascular volume and fluid responsiveness, especially in the context of
persistent lethargy. Urine output is also a good indicator of end-organ perfusion, but CVP
directly reflects preload.
45. A 30-year-old male presents to the emergency department with a scald burn after
accidentally spilling boiling water on himself. He has severe pain, redness, and blister
formation, especially on the palms of both hands and forearms. The affected area is estimated
to be 2% of total body surface area (TBSA). On examination: burns appear moist, red, and
have intact blisters, capillary refill is brisk in the affected areas, no signs of eschar or full-
thickness necrosis, normal distal pulses and no evidence of compartment syndrome. His
vitals are HR: 90 bpm, BP: 120/75 mmHg, RR: 18/min, SpO2: 98%. What is the most
appropriate next step in management?

Answer: D. Application of topical antibiotics and non-adherent dressing

Explanation: The patient has partial-thickness burns (blisters, moist, red) over a small TBSA
(2%), without signs of full-thickness injury or compromise to circulation. The immediate
management for such burns after ensuring ABCs and pain control involves cleaning the
wound and applying topical antibiotics (like silver sulfadiazine) and non-adherent dressings
to prevent infection and promote healing.

46. A 30-year-old pregnant woman at 26 weeks' gestation presents for routine antenatal
checkup. She reports a family history of diabetes mellitus in her mother and has noted
excessive thirst and frequent urination over the past few weeks. Her pre-pregnancy BMI was
28 kg/m². Her lab values are fasting blood glucose: 110 mg/dL and random blood glucose:
155 mg/dL. What is the best management approach for her condition?

Answer: B. Gestational Glucose Tolerance Test (GGTT)

Explanation: The patient is pregnant, obese (pre-pregnancy BMI 28), has a family history of
diabetes, and is reporting symptoms suggestive of hyperglycemia (excessive thirst, frequent
urination). These are risk factors and symptoms for Gestational Diabetes Mellitus (GDM).
The most appropriate next step for definitive diagnosis is a Gestational Glucose Tolerance
Test (GGTT), which typically involves a 1-hour or 2-hour glucose challenge.

47. A 28-year-old male is brought to the emergency department after a motorcycle accident.
He was not wearing protective gear and sustained blunt abdominal trauma.

Note: The question is cut off. However, based on the scenario, it would likely ask about the
initial assessment or management of blunt abdominal trauma.

49. A 32-year-old male is brought to the emergency department after a high-speed road
traffic accident (RTA). He is unresponsiveness with a GCS is 6/15 and shows signs of severe
hypovolemic shock. His vitals are BP: 70/40 mmHg, HR: 130 bpm, RR: 28/min and SpO2:
94% on oxygen. The emergency team attempts peripheral IV cannulation, but multiple
attempts fail due to collapsed veins. There is no immediate access for fluid resuscitation.
What is the most appropriate alternative approach for vascular access in this patient?

Answer: B. Intraosseous (IO) access


Explanation: In a critically ill trauma patient with severe hypovolemic shock where
peripheral intravenous access is difficult or impossible due to collapsed veins, Intraosseous
(IO) access is the most appropriate and rapid alternative for immediate vascular access to
administer fluids, blood products, and medications. It can be inserted quickly and provides
access to the systemic circulation effectively.

50. A 55-year-old male presents to the emergency department with acute swelling and pain in
his right knee for the past 2 days. He denies any recent trauma but reports difficulty in
walking due to the pain. He also has mild fever (38.1°C) and a history of type 2 diabetes. On
examination, right knee is swollen, erythematous, warm to touch and range of motion is
severely restricted due to pain. There is also moderate joint effusion but no signs of trauma or
skin wounds over the knee. What is the most appropriate next step in the management of this
patient?

Answer D. Urgent arthrocentesis with synovial fluid analysis

Explanation: The patient presents with acute, painful, swollen, warm, erythematous knee
with fever, consistent with septic arthritis. Given the patient's history of diabetes (a risk
factor for infection), this is a medical emergency that requires prompt treatment to prevent
joint destruction. The most appropriate initial management involves urgent aspiration of the
joint fluid for analysis (gram stain, cell count, culture), followed immediately by
initiation of empiric broad-spectrum antibiotics and pain management.

53. A 60-year-old male with a history of chronic kidney disease (CKD) and hypertension
presents to the emergency department with fatigue, muscle weakness, and palpitations. On
examination, he is alert and oriented and no signs of fluid overload. Lab results are serum
Potassium: 6.7 mmol/L, ECG show peaked T waves, prolonged PR interval, serum
creatinine: 4.5 mg/dL and bicarbonate level: 18 mmol/L. Which of the following is the most
appropriate next step in managing this patient's hyperkalemia?

Answer: A. Insulin and Dextrose

Explanation: The patient has severe hyperkalemia (K+ 6.7 mmol/L) with ECG changes
(peaked T waves, prolonged PR interval) indicating cardiotoxicity. Insulin with dextrose is
the most appropriate immediate treatment to shift potassium into cells, rapidly lowering
serum potassium and stabilizing the cardiac membrane (along with calcium gluconate, if
available, which acts more quickly to stabilize the membrane but doesn't lower K+). Dialysis
would be for definitive removal but is not the immediate first step for cardiac stability.

54. A 46-year-old female undergoes a laparoscopic cholecystectomy for symptomatic


gallstones. During the procedure, the surgeon notices that insufflation pressures are dropping
intermittently, making it difficult to maintain adequate pneumoperitoneum. The operative
field visualization is poor, and the abdominal wall appears to be collapsing. The surgical
team checks the CO2 tank, which is full, and confirms the insufflator settings are correct (12-
15 mmHg). What is the most likely cause of the inability to maintain insufflation pressure
and poor visualization?

A. Undiagnosed bowel perforation into the gastric contents into the peritoneal
cavity: While a bowel perforation would cause problems, it wouldn't directly
cause intermittent dropping of insufflation pressure and difficulty maintaining
pneumoperitoneum. It could lead to complications but isn't the primary reason
for a gas leak.

B. Disconnection between the insufflation line and the trocar: This is a highly
likely cause. A disconnected line would cause immediate loss of insufflation
pressure and pneumoperitoneum.

C. Intra-abdominal adhesions limiting space for pneumoperitoneum:


Adhesions can make initial insufflation difficult, but wouldn't typically cause
intermittent drops or inability to maintain pressure once established, unless
there's a specific tear.

D. Malfunction of the laparoscopic camera system: This affects visualization


but not insufflation pressure.

E. Incorrect gas mixture (e.g., using air instead of CO2): This would affect
visibility and possibly patient safety, but not directly cause intermittent
pressure drops or inability to maintain pneumoperitoneum in this manner.

Conclusion for Q54: Based on the classic signs, B. Disconnection between the
insufflation line and the trocar is the most direct and common cause for the
described problem.

55. A 45-year-old man presents to the emergency department with a history of a


puncture wound to his foot 3 days ago. He reports experiencing muscle spasms
in his jaw and neck, along with difficulty swallowing. On examination, he is
noted to have a rigid jaw, a fixed grimace, and hyperreflexia. What is the most
appropriate treatment?

Answer: D. Antitetanus immunoglobulin (TIG) and tetanus toxoid vaccine (TT)

Explanation: The patient has classic symptoms of tetanus (trismus/lockjaw, dysphagia,


muscle spasms, fixed grimace/risus sardonicus, hyperreflexia) following a puncture wound.
This is a severe, life-threatening condition. The most appropriate treatment involves both
passive immunization with Tetanus Immunoglobulin (TIG) to neutralize circulating toxin
and active immunization with the tetanus toxoid vaccine (TT). Additionally, supportive
care (sedation, muscle relaxants, airway management) and wound debridement are crucial.
57. A 25-year-old female underwent an ovarian cystectomy for a 5 cm endometrioma. She
had a history of dysmenorrhea, dyspareunia, and chronic pelvic pain prior to surgery. Now,
she is seeking advice on how to prevent recurrence of endometriosis and manage her ongoing
symptoms. Her pelvic exam is unremarkable. Which of the following is the most appropriate
treatment to help prevent recurrence of endometriosis and manage symptoms after surgery?

Answer: A. Combined oral contraceptive pills (COCPs)

Explanation: Endometriosis is a chronic condition characterized by estrogen-dependent


growth of endometrial tissue outside the uterus. After surgical removal of endometriomas,
hormonal suppression is often used to prevent recurrence and manage symptoms. Combined
oral contraceptive pills (COCPs) are a first-line medical therapy for endometriosis as they
suppress ovulation and endometrial growth, reducing pain and preventing lesion recurrence.

58. A 42-year-old female, who underwent a total abdominal hysterectomy one year ago for
fibroids, presents with complaints of passing air from the vagina. She denies any urinary
symptoms, fecal incontinence, or abdominal pain. On examination, abdomen is soft, non-
tender, normal external genitalia and no visible discharge. Which of the following is the most
appropriate diagnostic approach to determine the cause of this symptom?

Answer: B. CT scan of the abdomen and pelvis with contrast

Explanation: The patient's symptom of "passing air from the vagina" (pneumovagina) after a
hysterectomy suggests a possible fistula (e.g., rectovaginal fistula or colovaginal fistula).
While a physical exam might not reveal it externally, and simple ultrasound might be
inconclusive, a CT scan of the abdomen and pelvis with contrast is the most appropriate
initial diagnostic imaging to identify the presence and source of a fistula by showing contrast
extravasation into the vagina from the bowel.

59. A 48-year-old male with a history of episodic headaches, palpitations, and sweating is
diagnosed with pheochromocytoma after elevated plasma metanephrines and a positive
adrenal mass on imaging. He is scheduled for adrenalectomy and requires preoperative blood
pressure optimization to prevent intraoperative hypertensive crises. His vitals are blood
pressure: 180/100 mmHg and heart rate 92 bpm. On examination, there are no signs of heart
failure or volume depletion. Which of the following antihypertensive medications should be
given before surgery to optimize blood pressure and prevent intraoperative complications?

Answer: Phenoxybenzamine

Explanation: The patient has pheochromocytoma, a catecholamine-secreting tumor, and


requires preoperative alpha-blockade to control hypertension and prevent hypertensive crises
during surgery. Phenoxybenzamine is a non-selective, irreversible alpha-blocker that is the
drug of choice for initial blood pressure control in pheochromocytoma patients. Beta-
blockers (like propranolol) should only be added after adequate alpha-blockade has been
achieved to prevent unopposed alpha-constriction.
60. A 55-year-old man, who has been working in a fertilizer company for the past 20 years,
presents with a history of chronic smoking. He underwent urinalysis on two separate
occasions, both of which revealed microscopic hematuria. His physical examination is
unremarkable, with no other significant findings. Given his occupational exposure and
smoking history, what is the next best step in diagnosing the cause of his hematuria?

Answer: CT scan of the abdomen and pelvis with contrast

Explanation: The patient has painless microscopic hematuria with significant risk factors for
urological malignancy (smoking, occupational exposure to chemicals, age). A CT scan of the
abdomen and pelvis with contrast is the most appropriate initial diagnostic step to evaluate
for renal masses, ureteral abnormalities, and bladder lesions. Cystoscopy might follow if CT
is inconclusive or bladder pathology is suspected.

61. A 60-year-old woman presents to the emergency department with a red eye, nausea, and
vomiting. She also reports blurred vision and halos around lights. On examination, she had
eye is hyperemic, and she exhibits hypermetric saccades. She mentions that she has
experienced similar symptoms in the past, but they resolved spontaneously after a few hours.
She denies any recent trauma or infectious symptoms. What is the most likely diagnosis?

Answer: D. Acute Angle-Closure Glaucoma

Explanation: The constellation of symptoms—red eye, nausea, vomiting, blurred vision,


halos around lights, hyperemic eye, and a history of similar, self-resolving episodes—is
classic for an acute attack of angle-closure glaucoma. This condition is an ophthalmologic
emergency caused by a sudden increase in intraocular pressure.

62. A 35-year-old male is brought to the emergency department after a road traffic accident
(RTA). His blood pressure is 80/60 mmHg, he is oliguric, and his mental status is confused.
Based on these clinical signs, what is the approximate percentage of blood volume that has
been lost?

Answer: C. 30%

Explanation: The patient is in Class III hemorrhagic shock based on his vital signs (BP
80/60 mmHg, tachycardia) and clinical signs (oliguria, confused mental status). Class III
shock is characterized by an estimated blood loss of 30-40% of total blood volume
(approximately 1500-2000 mL in an adult).

63. A 3-year-old child presents to the emergency department with a 3-day history of
worsening respiratory symptoms. The child initially had a mild cough and hoarse voice, but
over the past 48 hours, the symptoms have progressed to include both inspiratory and
expiratory stridor and episodes of respiratory distress, particularly at night. The parents report
that the child had a low-grade fever initially, but it resolved yesterday. Despite supportive
care at home, the child's breathing difficulties have increased, and the child now appears
anxious and fatigued. On examination, the child feel comfort in supine position. What is the
most likely diagnosis?

Answer: B. Laryngotracheobronchitis

Explanation: The child's symptoms of initial mild cough and hoarse voice progressing to
inspiratory and expiratory stridor, respiratory distress worse at night, and a preceding low-
grade fever are highly characteristic of Laryngotracheobronchitis (Croup), which is
typically viral. The "comfort in supine position" is less specific but doesn't rule out croup.
Acute epiglottitis would present more acutely with high fever, drooling, and a toxic
appearance, often preferring to sit upright.

Surgery 2023

Question 5

A 40-year-old man presents to the clinic with severe epigastric pain that predominantly occurs at
night, often waking him from sleep. He describes the pain as burning and intermittent, and it is
relieved temporarily by eating. The patient is a long-term smoker with a 40-pack-year history
and admits to having a poor eating habit. He denies vomiting, significant weight loss, or changes
in bowel habits. Laboratory findings are within normal limits. What is the most likely diagnosis?

A. duodenal ulcer

B. Acute pancreatitis

C. Mesenteric ischemia

D. Inferior wall myocardial infarction

The correct answer is A. Duodenal ulcer

Explanation: The patient's symptoms are a classic presentation of a gastric ulcer. Key indicators
include nocturnal epigastric pain that wakes him from sleep, the burning nature of the pain, and
temporary relief with eating. The patient's history of smoking is a significant risk factor for ulcer
disease.
Question 6

A 70-year-old man with a known history of congestive heart failure (CHF) presents to the clinic
with worsening shortness of breath on exertion for the past two weeks. He denies chest pain,
palpitation, or fever. His symptoms are aggravated during his physical activity. On physical
examination, his vital signs are stable, and he is not in distress at rest. His bilateral lower limb
edema is present. His auscultation reveals crackles at the lung bases. Which investigation would
be the most appropriate to carry out?

A. Echocardiogram

B. Electrocardiogram (ECG)

C. Cardiac enzymes

D. Pulmonary function test (PFT)

E. Angiography

The correct answer is A. Echocardiogram.

Explanation: The patient has a history of CHF and is showing signs of worsening heart failure,
specifically fluid overload (lower limb edema and crackles in the lungs) and shortness of breath
on exertion. An echocardiogram is the most appropriate investigation because it can assess the
heart's pumping function, valve integrity, and overall structure, which are crucial for managing
and understanding the progression of heart failure.

Question 7

A 28-year-old married woman presents to the clinic with a complaint of 5 weeks of amenorrhea.
Her menstrual cycles have been regular in the past, and she is sexually active without using
contraception. She reports mild fatigue and breast tenderness but denies any abdominal pain or
vaginal bleeding. On examination, she appears well, and no abnormalities are noted. What is the
best test at this time to confirm her pregnancy?

A. Serum B-HCG

B. Urine B-HCG

C. Transabdominal ultrasound

D. Transvaginal ultrasound

E. Progesterone level

The correct answer is A. Serum B-HCG.


Explanation: The patient presents with symptoms and a history highly suggestive of pregnancy
(amenorrhea, fatigue, breast tenderness, unprotected sexual activity). A serum B-HCG test is the
most definitive and sensitive blood test to confirm pregnancy. While a urine test can also confirm
pregnancy, the serum test provides a quantitative value, which can be helpful in monitoring the
pregnancy.

Question 8

A 28-year-old man presents to the clinic with complaints of painful urination that started two
weeks ago, swelling and pain in his right ankle, and redness in both eyes accompanied by
discharge. He denies recent trauma or systemic symptoms such as fever or weight loss. His past
medical history is unremarkable, but he reports a recent episode of unprotected sexual
intercourse one month ago. What is the most appropriate initial management for this patient?

A. Ceftriaxone and Azithromycin

B. Acyclovir

C. Prednisone

D. Allopurinol

E. NSAIDs

The correct answer is A. Ceftriaxone and Azithromycin.

Explanation: This patient's presentation of painful urination, arthritis in a single joint (right
ankle), and conjunctivitis (redness in both eyes with discharge) following unprotected sexual
intercourse is a classic triad for Reiter's syndrome, also known as reactive arthritis. This
condition is often triggered by a bacterial infection, most commonly Chlamydia trachomatis or
Neisseria gonorrhoeae. The most appropriate initial management is to treat the underlying
infection with a combination of antibiotics, such as Ceftriaxone and Azithromycin, to cover for
both gonorrhea and chlamydia.

While the image is still somewhat blurry, I can make out the questions and provide the most
likely answers with explanations.

Question 17

A 66-year-old male presents to the emergency department with a 5-day history of fever, pleuritic
chest pain, and worsening shortness of breath. He has a temperature of 38.5°C. On examination,
he has decreased air entry on the right lower lung base, and there is dullness to percussion in the
same area. A chest x-ray is performed which is shown below. What is the most likely diagnosis?
A. Parapneumonic effusion

B. Lung abscess

C. Empyema

D. Pleural effusion

E. Pulmonary embolism

The correct answer is A. Parapneumonic effusion.

Explanation: The patient's symptoms of fever, pleuritic chest pain, decreased breath sounds, and
dullness to percussion are classic signs of a pneumonia with a fluid collection in the pleural
space. A parapneumonic effusion is a pleural effusion that occurs as a complication of
pneumonia.

Question 18

A 28-year-old patient is brought to the emergency department following a serious accident


involving head trauma and a pelvic fracture. The patient is in critical condition, and measures are
being taken to prevent secondary complications. Which of the following factors should be
avoided to minimize the risk of ischemic injury to the fractured basin?

A. Hyperglycemia

B. Hyperthermia

C. Hypertension

D. Hypotension

E. Hypothermia

The correct answer is D. Hypotension.

Explanation: Ischemic injury is damage caused by a lack of blood supply. In a patient with a
pelvic fracture, maintaining adequate blood pressure is crucial. Hypotension (low blood
pressure) directly reduces blood flow to the injured tissues, increasing the risk of ischemia and
further tissue damage. Therefore, it must be avoided.

Question 19

A 32-year-old patient arrives at the emergency department with a stab wound in the 5th
intercostal space (ICS), 5 cm from the left sternal border. The patient is tachycardic with a pulse
rate of 120 beats per minute and hypotensive with a blood pressure of 90/60 mmHg. The patient
also reports shortness of breath, and breath sounds are absent on the affected side. What is the
best initial step in management?

A. Chest tube insertion

B. Pericardiocentesis

C. Open thoracotomy

D. Needle decompression

E. Immediate intubation

The correct answer is D. Needle decompression.

Explanation: The patient's presentation of a chest wound, tachycardia, hypotension, and absent
breath sounds is a classic sign of a tension pneumothorax. This is a life-threatening condition
where air builds up in the chest cavity, collapsing the lung and putting pressure on the heart and
major blood vessels. The most immediate, life-saving intervention is needle decompression to
release the trapped air and restore circulation.

Question 20

A 7-year-old child is brought to the emergency department after falling from stairs. The child
landed on their outstretched hand and is now complaining of severe pain in their right elbow and
forearm. On examination, there is obvious deformity and significant swelling around the elbow
joint and forearm. The child is unable to move the elbow, and there is severe pain on passive
movement of the fingers and wrist. Additionally, there is decreased sensation in the hand, a weak
radial pulse, and the skin of the forearm appears tense and shiny. What is the most likely
diagnosis?

A. Fracture of the humerus with compartment syndrome

B. Fracture of the humerus without compartment syndrome

C. Fracture of the forearm with compartment syndrome

D. Laceration of the brachial artery

E. Isolated supracondylar fracture without complications

The correct answer is A. Fracture of the humerus with compartment syndrome.

Explanation: The patient's clinical picture, particularly the "5 P's" of compartment syndrome
(pain, pallor, paresthesia, pulselessness, and poikilothermia), is evident here with severe pain on
passive movement, decreased sensation, and a weak radial pulse. A fall on an outstretched
hand in a child is a very common cause of a supracondylar fracture of the humerus, which is
notorious for causing this severe and dangerous complication.

Question 21

A 45-year-old individual presents to the emergency department following a motor vehicle


accident resulting in a pelvic fracture. Examination reveals blood at the urethral meatus and a
palpable, distended bladder. The patient complains of an inability to pass urine. There is no
evidence of gross hematuria or perineal hematoma. What is the most appropriate initial
management for this patient?

A. Cystostomy

B. Urethral catheterization

C. Suprapubic cystostomy

D. IV antibiotics

E. Pelvic binder

The correct answer is C. Suprapubic cystostomy.

Explanation: The combination of a pelvic fracture, blood at the urethral meatus, and a distended
bladder is highly suggestive of a urethral injury. In this situation, attempting to pass a urethral
catheter is contraindicated as it could worsen the injury. The most appropriate management is to
decompress the bladder by placing a catheter directly into the bladder through the abdominal
wall, a procedure known as a suprapubic cystostomy.

Question 22

A 59-year-old traveler who recently returned from Chicago to Karachi presents to the clinic with
acute onset of watery diarrhea. The patient reports multiple episodes of stools. There is no
history of fever, vomiting, or significant dehydration. The patient and his family ate different
foods during the trip. What is the most likely causative organism for the diarrhea?

A. Giardia

B. Shigella

C. Vibrio cholera

D. Escherichia coli
E. Amebiasis

The correct answer is D. Escherichia coli.

Explanation: This patient's symptoms are classic for traveler's diarrhea, which is a common
illness affecting travelers to developing countries. The most frequent cause is enterotoxigenic E.
coli (ETEC), which produces a toxin that causes watery diarrhea without fever or significant
systemic symptoms.

Question 23

A 24-year-old female presents to the clinic with recurring episodes of tachycardia, sweating,
shortness of breath, and a feeling of dread whenever she visits crowded places such as markets or
malls. She reports that these symptoms begin abruptly and are accompanied by a fear of losing
control or being unable to escape the situation. These episodes have led her to avoid going to
crowded areas altogether. Based on her presentation, what is the most likely diagnosis?

A. Agoraphobia

B. Social Anxiety Disorder

C. Panic Disorder

D. Generalized Anxiety Disorder

E. Obsessive-Compulsive Disorder

The correct answer is C. Panic Disorder.

Explanation: The patient is experiencing recurrent, unexpected episodes of intense fear (panic
attacks) with physical symptoms like tachycardia and shortness of breath. Her fear of having
another attack and the resulting avoidance of certain places (crowded markets or malls) is
characteristic of Panic Disorder with Agoraphobia. The core diagnosis, however, is the Panic
Disorder itself.

Question 24

A 45-year-old male presents to the urology clinic with complaints of a painless swelling in his
right testicle that he noticed a few weeks ago. Upon examination, a 2 cm mass is palpable within
the right testicle. An ultrasound confirms the presence of a testicular tumor. Further
investigations, including a CT scan, reveal the presence of metastases. The tumor has spread to
regional lymph nodes. Which of the following lymph nodes is most likely to be involved in
draining the site of the testicular tumor?

A. Axillary lymph nodes


B. Inguinal lymph nodes

C. Femoral lymph nodes

D. Popliteal lymph nodes

E. Para-aortic lymph nodes

The correct answer is E. Para-aortic lymph nodes.

Explanation: The lymphatic drainage of the testicles follows the testicular blood vessels, which
originate from the retroperitoneum. Therefore, the primary regional lymph nodes for testicular
cancer are the para-aortic lymph nodes, which are located along the aorta in the posterior
abdominal wall.

Question 25

A 60-year-old male patient, with a history of previous inguinal hernia repair, presents to the
emergency department with a sudden onset of severe pain and swelling in the right inguinal
region. He reports nausea and vomiting but denies fever or systemic symptoms. On examination,
the swelling is irreducible but is not tender. The skin of the inguinal region is reddened and
warm. The abdomen reveals air-fluid levels. Laboratory findings are normal. What is the most
definitive treatment for this patient?

A. Open Mesh repair via suprapubic incision

B. Laparoscopic repair

C. Observational management

D. Manual reduction

E. Emergency surgery with hernia repair

The correct answer is E. Emergency surgery with hernia repair.

Explanation: The patient's symptoms of an irreducible and painful inguinal swelling, along with
signs of bowel obstruction (nausea, vomiting, and air-fluid levels on imaging), indicate an
incarcerated or strangulated hernia. This is a surgical emergency that requires prompt
intervention to prevent bowel necrosis. Therefore, the definitive treatment is emergency surgery
with hernia repair.
Question 26

A 42-year-old businessman, who frequently travels for work, presents to the proctology clinic
with a history of recurrent episodes of painful swelling around the anus for the past several
months. He describes the pain as throbbing and severe, often becoming unbearable. He reports
that the pain is typically relieved after a yellowish discharge occurs. On a further questioning, he
denies any visible external wounds, bleeding, or changes in bowel habits. He has no history of
fever, weight loss, or systemic symptoms. Digital rectal examination reveals tenderness in the
anal region but no palpable masses or fissures. What is the most likely diagnosis for this patient's
condition?

A. Fistula-in-Ano

B. Perianal Abscess

C. Hemorrhoids

D. Anal Fissure

E. Pilonidal Sinus

The correct answer is B. Perianal Abscess.

Explanation: The patient's history of recurrent painful swelling around the anus that
spontaneously drains a yellowish discharge is a classic sign of a perianal abscess. The cycle of
swelling, pain, and eventual rupture is characteristic of this condition. A fistula-in-ano often
develops after an abscess, but the primary symptom described is the abscess itself.

Question 27

A 41-year-old woman presents to the breast clinic with a recently noticed mass in her right
breast. She reports that the mass is painless but feels hard and does not change in size with her
menstrual cycle. On clinical examination, a fixed, hard, and irregular mass is palpated in the
upper outer quadrant of the right breast. The mass is not adherent to the underlying chest wall or
skin, and there is no associated nipple discharge, skin changes, or axillary lymphadenopathy. The
patient has no significant family history of breast cancer and no prior history of breast disease.
What is the most appropriate next step for investigation?

A. Fine Needle Aspiration Cytology (FNAC)

B. Excisional biopsy

C. Ultrasound of Breast

D. Mammogram

E. Clinical Observation and follow-up

The correct answer is D. Mammogram.


Explanation: The patient's mass has several features concerning for malignancy: it is hard,
fixed, and irregular. A mammogram is the most appropriate initial imaging investigation for a
woman over 40 with a suspicious breast mass. It is a crucial step to further characterize the lesion
and guide the need for subsequent biopsy.

Question 28

A 6-year-old child is brought to the pediatric clinic with complaints of itching around the anus,
particularly at night. The parents report that the child has been consuming a diet high in sugar,
including 3 chocolates bars and 12 ounces of milk daily. They also mention that the child
occasionally eats their underwear, with small amounts of stool. The child has no history of fever,
weight loss, or abdominal pain. On examination, there is mild perianal erythema but no visible
fissures, masses, or signs of infection. What is the most likely condition contributing to the
child's symptoms?

A. Anal fissure

B. Scabies (Enterobius)

C. Lactose intolerance

D. Constipation

E. Pinworm Infestation

The correct answer is E. Pinworm Infestation.

Explanation: The cardinal symptom of perianal itching at night in a child is a textbook


presentation of a pinworm (Enterobius vermicularis) infestation. The itching occurs as the
female pinworm migrates to the perianal area to lay her eggs, typically at night.

Question 29

A 58-year-old male patient with a history of hypertension presents to the primary care clinic with
complaints of sudden onset of excruciating pain, swelling, and warmth in his right big toe. He
describes the pain as severe and mentions that the affected joint is tender to touch. The patient
also reports that he has been experiencing episodes of elevated blood pressure despite being on
antihypertensive medications. On further inquiry, it is revealed that he is currently taking
medications for hypertension management. Which of the following medications might be
contributing to the development of his current symptoms?

A. Beta-blockers

B. ACE inhibitors
C. Thiazide diuretics

D. Calcium Channel Blockers

E. Angiotensin II Receptor Blockers

The correct answer is C. Thiazide diuretics.

Explanation: The patient's symptoms of sudden onset of severe pain, swelling, and warmth in
the big toe (podagra) are highly characteristic of an acute gout attack. Thiazide diuretics are
known to increase serum uric acid levels, which can precipitate a gout flare.

Question 30

A 64-year-old male patient, with a 30-year history of tobacco use, presents to the oncology clinic
with complaints of persistent hoarseness, difficulty swallowing (dysphagia), and a feeling of a
lump in his throat. On examination, a fungating growth is observed in the laryngeal region. The
patient also reports unintentional weight loss and fatigue over the past few months. He has no
significant history of alcohol use or other risk factors. Which of the following is the most likely
diagnosis?

A. Laryngotracheitis

B. Squamous Cell Carcinoma

C. Small Cell Carcinoma

D. Lymphoma

E. Neuroendocrine Tumor

The correct answer is B. Squamous Cell Carcinoma.

Explanation: This is a classic presentation of laryngeal cancer. The patient's long-standing


history of tobacco use is a major risk factor. The symptoms of persistent hoarseness,
dysphagia, and a lump in the throat are hallmarks of a laryngeal tumor, and the most common
histological type is Squamous Cell Carcinoma.

Question 31

A 22-year-old African-American male presents to the emergency department with severe pain in
both legs and arms for the past 12 hours. The pain is described as sharp and constant, worsening
with movement. He has a history of similar painful episodes requiring hospitalization. He denies
recent trauma or fever. On examination, he appears in distress with tenderness over multiple
bony areas but no obvious swelling or erythema. Laboratory investigations reveal a hemoglobin
of 8.2 g/dL. Hematoglobin electrophoresis reveals an abnormal pattern, and peripheral blood
smear is shown in the diagram. What is the most likely diagnosis?

A. Thalassemia

B. Iron Deficiency Anemia

C. Sickle Cell Disease

D. Autoimmune hemolytic Anemia

E. Hereditary Spherocytosis

The correct answer is C. Sickle Cell Disease.

Explanation: The patient's presentation of a vaso-occlusive crisis (severe, painful episodes in


the limbs) in a young African-American male with a history of similar hospitalizations is highly
suggestive of Sickle Cell Disease. The laboratory findings of a low hemoglobin count and an
abnormal hemoglobin electrophoresis pattern further confirm this diagnosis.

Question 32

A 36-year-old female patient presents to the emergency department with episodic spells of
severe headache, palpitations, and profuse sweating. She describes these episodes as sudden in
onset, lasting for several minutes, and accompanied by feelings of anxiety and shakiness. During
these episodes, her blood pressure spikes significantly. Physical examination reveals elevated
blood pressure and a rapid heart rate during an episode. She has no significant past medical
history and is not taking any medications. What is the most appropriate diagnostic test to confirm
the suspected condition in this patient?

A. Serum Catecholamines

B. 24-hour Urinary Vanillylmandelic Acid

C. 24-hour Urinary Metanephrines

D. Serum Cortisol

E. Serum Thyroid Hormones

The correct answer is C. 24-hour Urinary Metanephrines.

Explanation: The patient's symptoms of episodic severe headaches, palpitations, and profuse
sweating, combined with sudden spikes in blood pressure, form the classic triad for a
pheochromocytoma. This is a tumor of the adrenal gland that secretes excess catecholamines.
The most reliable diagnostic test to confirm this is a 24-hour urinary collection for
metanephrines and normetanephrines, which are the breakdown products of catecholamines.

Question 33

A 38-year-old female patient visits her primary care physician with complaints of joint pain and
stiffness. She mentions experiencing significant morning stiffness lasting more than 40 minutes,
which is most pronounced in her hands and wrists, making it difficult for her to button her
clothes or perform other fine motor tasks. The pain and stiffness improve throughout the day
with activity. On physical examination, there is symmetrical swelling and tenderness in the
metacarpophalangeal (MCP) and proximal interphalangeal (PIP) joints of both hands. Which of
the following is the most likely diagnosis?

A. Osteoarthritis

B. Rheumatoid Arthritis

C. Systemic Lupus Erythematosus

D. Gout

E. Psoriatic Arthritis

The correct answer is B. Rheumatoid Arthritis.

Explanation: This is a classic presentation of Rheumatoid Arthritis. The key features are
symmetrical involvement of small joints, particularly the MCP and PIP joints of the hands,
and morning stiffness lasting for more than 30 minutes that improves with activity.

I can answer all the questions from the images you've provided, including the question text itself.

Question 5

A 40-year-old man presents to the clinic with severe epigastric pain that predominantly occurs at
night, often waking him from sleep. He describes the pain as burning and intermittent, and it is
relieved temporarily by eating. The patient is a long-term smoker with a 40-pack-year history
and admits to having a poor eating habit. He denies vomiting, significant weight loss, or changes
in bowel habits. Laboratory findings are within normal limits. What is the most likely diagnosis?

A. Duodenal ulcer

B. Acute pancreatitis

C. Mesenteric ischemia
D. Inferior wall myocardial infarction

The correct answer is Duodenal ulcer.

Justification: The patient's symptoms of nocturnal epigastric pain, a burning sensation, and
temporary relief with food are classic signs of a duodenal ulcer. His long history of smoking is a
significant risk factor. Triggered by eating is gastric ulcer, relieved by eating is duodenal.
Question 6

A 70-year-old man with a known history of congestive heart failure (CHF) presents to the clinic
with worsening shortness of breath on exertion for the past two weeks. He denies chest pain,
palpitation, or fever. His symptoms are aggravated during his physical activity. On physical
examination, his vital signs are stable, and he is not in distress at rest. His bilateral lower limb
edema is present. His auscultation reveals crackles at the lung bases. Which investigation would
be the most appropriate to carry out?

A. Echocardiogram

B. Electrocardiogram (ECG)

C. Cardiac enzymes

D. Pulmonary function test (PFT)

E. Angiography

The correct answer is Echocardiogram.

Justification: The patient is showing signs of worsening heart failure (fluid overload evidenced
by pedal edema and rales). An echocardiogram is the best test to visualize the heart's structure
and function, which is critical for managing CHF.
Question 7

A 28-year-old married woman presents to the clinic with a complaint of 5 weeks of amenorrhea.
Her menstrual cycles have been regular in the past, and she is sexually active without using
contraception. She reports mild fatigue and breast tenderness but denies any abdominal pain or
vaginal bleeding. On examination, she appears well, and no abnormalities are noted. What is the
best test at this time to confirm her pregnancy?

A. Serum B-HCG

B. Urine B-HCG

C. Transabdominal ultrasound
D. Transvaginal ultrasound

E. Progesterone level

The correct answer is Serum B-HCG.

Justification: A serum B-HCG test is the most definitive and sensitive method to confirm a
pregnancy, especially in the early stages.
Question 8

A 28-year-old man presents to the clinic with complaints of painful urination that started two
weeks ago, swelling and pain in his right ankle, and redness in both eyes accompanied by
discharge. He denies recent trauma or fever. His past medical history is unremarkable, but he
reports a recent episode of unprotected sexual intercourse one month ago. What is the most
appropriate initial management for this patient?

A. Ceftriaxone and Azithromycin

B. Acyclovir

C. Prednisone

D. Allopurinol

E. NSAIDs

The correct answer is Ceftriaxone and Azithromycin.

Justification: The triad of urethritis, arthritis, and conjunctivitis following unprotected sexual
intercourse is characteristic of reactive arthritis REITERS SYNDROME, often caused by a
chlamydial or gonococcal infection. Treatment involves a combination of antibiotics to cover
both organisms.

Question 34

A 38-year-old female patient visits her primary care physician with complaints of joint pain and
stiffness. She mentions experiencing significant morning stiffness lasting more than 40 minutes,
which is most pronounced in her hands and wrists, making it difficult for her to button her
clothes or perform other fine motor tasks. The pain and stiffness improve throughout the day
with activity. On physical examination, there is symmetrical swelling and tenderness in the
metacarpophalangeal (MCP) and proximal interphalangeal (PIP) joints of both hands. Which of
the following is the most likely diagnosis?

A. Osteoarthritis
B. Rheumatoid Arthritis

C. Systemic Lupus Erythematosus

D. Gout

E. Psoriatic Arthritis

The correct answer is Rheumatoid Arthritis.

Justification: The key features of symmetrical joint involvement of the small joints of the
hands and morning stiffness lasting over 30 minutes that improves with activity are classic
signs of rheumatoid arthritis.
Question 35

A 4-year-old child is brought to the pediatric clinic with symptoms of fever, cough, and rapid
and difficult breathing, accompanied by chest retractions. On examination, the pediatrician
observes signs of respiratory distress, including increased effort in breathing and visible chest
retractions. Based on the Integrated Management of Newborn and Childhood Illness (IMNCI)
classification, which category does this child's condition fall under?

A. Uncomplicated Pneumonia

B. Very severe disease

C. Severe Pneumonia

D. No Pneumonia

E. Mild Pneumonia

The correct answer is Severe Pneumonia.

Justification: According to the IMNCI guidelines, a child with cough or difficulty breathing
plus chest indrawing (retractions) or stridor is classified as having severe pneumonia.
Question 36

A 59-year-old male patient with a history of smoking and rheumatoid arthritis presents to the
emergency department with sudden-onset severe epigastric pain that started shortly after eating.
The pain was initially localized but has now become diffuse across the abdomen. He also reports
nausea and multiple episodes of vomiting. His past medical history is significant for long-term
NSAID use for arthritis. On examination, he appears pale, diaphoretic, and in distress. His blood
pressure is 85/50 mmHg, heart rate 118 bpm, and respiratory rate 24 breaths per minute. His
abdomen is rigid with generalized tenderness and absent bowel sounds. An upright chest X-ray is
shown below. What is the most likely diagnosis?
A. Acute Pancreatitis

B. Acute Cholecystitis

C. Small Bowel Obstruction

D. Perforated Peptic Ulcer

E. Ruptured Abdominal Aortic Aneurysm

The correct answer is Perforated Peptic Ulcer.

Justification: The patient's history of long-term NSAID use, a classic risk factor, combined with
the sudden onset of severe, diffuse abdominal pain, a rigid abdomen, and signs of shock
(hypotension, tachycardia) is highly suggestive of a perforated peptic ulcer.
Question 37

A 45-year-old male patient presents to the emergency department with sudden-onset severe
upper abdominal pain that radiates to the back. The pain is burning in nature and most
pronounced shortly after eating. He reports occasional nausea but denies vomiting. He has a
history of chronic smoking, NSAID use for arthritis, and occasional alcohol consumption. On
physical examination, he has marked epigastric tenderness but no rebound tenderness or
guarding. His vital signs are stable. Laboratory tests show a mild drop in hemoglobin. A double-
contrast CT scan with contrast reveals a posterior duodenal ulcer with adjacent vascular
involvement. Which of the following arteries is most likely affected in this patient?

A. Celiac Artery

B. Gastroduodenal Artery

C. Inferior Pancreaticoduodenal Artery

D. Left Gastric Artery

E. Splenic Artery

The correct answer is Gastroduodenal Artery.

Justification: A duodenal ulcer that is posterior is at risk of eroding into the gastroduodenal
artery, which lies directly behind the duodenum. This is a common cause of severe upper
gastrointestinal bleeding.
Question 38

A 35-year-old multiparous woman (G5, P4+1) presents to the clinic with complaints of fatigue,
generalized weakness, and shortness of breath that have progressively worsened over the past
several months. She reports heavy menstrual bleeding and poor dietary intake. On examination,
she appears pale, and her conjunctiva is also pale. Given her symptoms and clinical findings,
which of the following is the most appropriate initial investigation?

A. Myoglobin

B. Total Iron Binding Capacity (TIBC)

C. Serum Ferritin

D. Serum B12

E. Hemoglobin Electrophoresis

Question 39

A 54-year-old patient previously underwent surgery to remove a tumor located in the ileocecal
region. The surgery was successful, and the patient had been in remission. However, the patient
now presents with concerning neurological symptoms. On examination, the healthcare provider
observes that the patient has lost the ability to sense vibrations and position in their lower limbs.
There is also diminished proprioception and touch sensation. Considering the patient's history
and current symptoms, what is the most likely cause of these neurological deficits?

A. Spinal Cord Compression

B. Vitamin B12 Deficiency

C. Peripheral Neuropathy

D. Multiple Sclerosis

E. Subarachnoid Hemorrhage

Justification:The ileocecal region is a common site for vitamin B12


absorption due to the presence of the intrinsic factor, which is necessary for
B12 absorption. Surgical removal of this region can impair B12 absorption,
leading to a deficiency. The neurological symptoms, particularly the loss of
vibration and position sense in the lower limbs, are hallmarks of vitamin B12
deficiency.
In summary, the patient's history, combined with the specific sensory loss
pattern, strongly points towards a vitamin B12 deficiency as the most likely
cause of the neurological deficits
Question 40

A 32-year-old woman presents to her primary care physician with a history of recurrent episodes
of cough, wheezing, and shortness of breath. She reports experiencing these symptoms more than
twice a week, with frequent nighttime awakenings that disrupt her sleep. The symptoms are
interfering with her daily activities. On physical examination, mild wheezing is noted on
auscultation. Spirometry reveals a moderate reduction in forced expiratory volume in one second
(FEV1), with significant improvement after bronchodilator administration. Given her clinical
presentation and objective findings, what is the most appropriate initial controller therapy for this
patient?

A. Inhaled Corticosteroid (ICS)

B. Long-Acting Beta Agonist (LABA) Monotherapy

C. Leukotriene Receptor Antagonist (LTRA)

D. Short-Acting Beta Agonist (SABA) alone

E. Combined Inhaled Corticosteroid and Long-Acting Beta Agonist (ICS+LABA)

The correct answer is E

Justification: The patient's symptoms (recurrent cough and wheezing, nighttime awakenings,
and FEV1 reduction) indicate moderately persistent asthma. For this level of severity, the most
appropriate initial controller therapy is an inhaled corticosteroid (ICS)+ LABA .

Question 41

Question: A 58-year-old male with a history of recurrent diverticulitis presents to the emergency
department with progressively worsening lower abdominal pain, fever, and a feeling of pelvic
fullness for the past three days. He has a decreased appetite and malaise but denies nausea or
vomiting. On examination, he appears ill and febrile, with localized tenderness in the lower
abdomen but no signs of peritonitis. Laboratory tests show leukocytosis and elevated C-reactive
protein (CRP). A CT scan of the abdomen and pelvis reveals a well-defined pelvic abscess
measuring 4 cm in diameter. Which of the following is the most appropriate initial management
for this patient?

The correct answer is CT-guided Drainage with IV Antibiotics.

Justification: The patient has a clear diagnosis of a diverticular abscess based on the CT scan.
The most appropriate initial management for an abscess of this size is a combination of
percutaneous drainage (CT-guided) and IV antibiotics to treat the underlying infection.
Question 42

Question: A 42-year-old male patient visits his primary care physician for a routine check-up.
His recent blood work reveals slightly elevated lipid levels. His triglyceride level is measured at
200 mg/dL, LDL cholesterol is 145 mg/dL, and HDL cholesterol is 40 mg/dL. The patient has no
significant medical history but he admits to occasionally indulging in unhealthy eating habits.
Based on his lipid profile, what is the most appropriate initial management strategy for this
patient?

 Total Cholesterol: Should be less than 200 mg/dL.


 LDL Cholesterol (Bad Cholesterol): Should be below 100 mg/dL, or below 70 mg/dL
for individuals with diabetes or heart disease.
 HDL Cholesterol (Good Cholesterol): Should be above 40 mg/dL, with higher levels
generally being better.
 Triglycerides: Should be below 150 mg/dL.

44. The most likely issue affecting the elderly mother's vision
is Cataracts. While a pinhole test can improve vision in cases of refractive
errors, it does not help with vision loss caused by cataracts. Cataracts, which
are a clouding of the lens, often develop gradually with age and cause blurred
or cloudy vision, making it difficult to read, watch TV, or perform other daily
tasks, according to the Cleveland Clinic.
Here's why the other options are less likely:
 Refractive Error:
Pinhole tests are designed to improve vision in cases of refractive errors (like
nearsightedness or farsightedness) by creating a smaller aperture that allows light to
focus more clearly on the retina. If a pinhole test doesn't improve vision, it's unlikely to
be the primary cause of the vision loss.
 Glaucoma:
Glaucoma damages the optic nerve, often leading to peripheral vision loss, not central
vision loss.
 Age-Related Macular Degeneration (AMD):
While AMD also causes central vision loss, it typically affects central vision more than
peripheral vision. Also, the pinhole test would not improve vision with AMD.
 Diabetic Retinopathy:
Diabetic retinopathy is a complication of diabetes that affects the blood vessels in the
retina. While it can cause blurred vision, it's more likely to cause blurred vision in
someone who has been diagnosed with diabete

Question 46

Question: A 45-year-old woman visits the surgical clinic due to concerns about her family
history of colon cancer. She wants to take proactive measures to assess her risk and ensure early
detection. She has no significant past medical history and has not undergone any colorectal
cancer screening. Given her family history, which of the following is the most appropriate
screening test for this patient?

The correct answer is Colonoscopy.

Question 47

Question: A 4-year-old child is brought to the pediatric clinic with symptoms of fever, cough,
and rapid and difficult breathing, accompanied by chest retractions. On examination, the
pediatrician observes signs of respiratory distress, including increased effort in breathing and
visible chest retractions. Based on the Integrated Management of Newborn and Childhood Illness
(IMNCI) classification, which category does this child's condition fall under?

The correct answer is Digital Manual Examination.

Justification: The patient is a female who is complaining of constipation with a feeling of


incomplete evacuation. A digital manual examination of the rectum is the most appropriate
step to assess the patient for fecal impaction, which is often the cause of this condition.
Question 48

Question 49

Question: A 50-year-old male presents to the emergency department with sudden and severe pain
in the upper right quadrant of her abdomen. She describes the pain as excruciating and constant,
radiating towards her right shoulder. The pain started after she had a fatty meal. She is nauseous
but hasn't vomited. On examination, her abdomen is tender to touch in the right upper quadrant.
Her vital signs are stable. What is the most likely cause of her symptoms?

The most likely cause of the woman's symptoms is B. Biliary Colic.


Explanation:
 Biliary Colic:
This condition is caused by a blockage of the bile duct (usually by a gallstone). The pain is
typically sudden, severe, and located in the right upper quadrant of the abdomen. The fact that
the pain started after a fatty meal, along with the nausea and shoulder radiation, strongly points
towards biliary colic.
 Acute Pancreatitis:
While pancreatitis can cause severe abdominal pain, it usually presents with pain radiating to
the back, which isn't mentioned in the scenario. The absence of vomiting and the stable vital
signs make pancreatitis less likely.
 Peptic Ulcer Disease:
Peptic ulcers typically cause epigastric pain (upper middle abdomen) rather than pain in the
right upper quadrant.
 Diverticulitis:
This condition usually causes pain in the lower left quadrant of the abdomen.
 Appendicitis:
Appendicitis usually presents with pain that starts near the belly button and then shifts to the
lower right quadrant.

Question 50

Question: A 28-year-old unmarried woman seeks psychological counseling due to overwhelming


feelings of anxiety and fear of being alone. She describes her relationship with her mother as
incredibly close and reveals that she cannot imagine functioning independently. She denies ever
being in a distance from her mother about her decisions and always even for mundane everyday
tasks. The thought of her mother leaving is passing away fills her with intense distress and panic.
She avoids making any significant life choices on her own, fearing that she might make the
wrong decision. Despite her intelligence and capabilities, she finds herself unable to take the
initiative or undertake responsibilities without her mother's presence and guidance. What
personality disorder is most likely underlying her behavior?

A. Borderline Personality Disorder

B. Avoidant Personality Disorder

C. Dependent Personality Disorder

D. Obsessive-Compulsive Personality Disorder

E. Narcissistic Personality Disorder

The correct answer is C. Dependent Personality Disorder.


Justification: The patient's behaviors of seeking constant reassurance, being unable to function
independently, and a profound fear of being alone or abandoned are the core features of
Dependent Personality Disorder.

Question 51

Question: A 51-year-old patient is brought to the emergency department following a road traffic
accident (RTA) with significant head trauma. The patient is unconscious and has difficulty
breathing. Given the critical nature of the situation, what is the most immediate and essential step
in managing this patient?

A. Administer Pure Oxygen

B. Perform Neurological Examination

C. Secure the Airway

D. Order a CT Scan of the Head

E. Start Intravenous Fluids

The correct answer is C. Secure the Airway.

Justification: In any trauma patient, the first priority is to secure the airway, followed by
breathing and circulation (the "ABC"s). With significant head trauma and difficulty breathing,
securing the airway is the most immediate and essential life-saving step.

Question 52

Question: A 32-year-old male with a known history of type 1 diabetes mellitus presents to the
emergency department with complaints of excessive thirst, frequent urination, fruity-smelling
breath, and abdominal pain. He admits to non-compliance with his insulin therapy for the last
two weeks due to personal stressors. On examination, he appears lethargic and dehydrated, with
vital signs showing tachycardia and a temperature of 100.4°F. Which of the following is the most
initial investigation for the diagnosis?

A. ABGs

B. Blood Glucose Measurement

C. Serum electrolytes

D. HbA1c

E. Ketone levels
The correct answer is B. Blood Glucose Measurement.

Justification: The patient's symptoms of excessive thirst, urination, and fruity-smelling breath,
combined with a history of non-compliance with insulin, are a classic presentation of diabetic
ketoacidosis. The most immediate and essential diagnostic step is a blood glucose measurement
to confirm the high blood sugar levels.

Question 53

Question: A 28-year-old breastfeeding woman presents to the clinic with a painful, swollen, and
erythematous lump in her left breast. She reports fever and chills for the past two days. On
examination, there is localized tenderness and warmth in the left breast with a small, palpable
fluctuant area (2 cm in diameter). There is no nipple discharge or skin changes. Based on these
findings, what is the most appropriate treatment for this patient's condition?

A. Warm compresses and observation

B. Oral antibiotics alone

C. Incision and drainage plus co-amoxiclav

D. Needle aspiration plus co-amoxiclav

E. CT-guided aspiration

The correct answer is D. Needle aspiration plus co-amoxiclav.

Justification: The patient's symptoms of a painful, swollen breast with fever and a palpable
fluctuant mass are characteristic of a breast abscess. The most appropriate management is to
perform a needle aspiration to drain the pus and administer oral antibiotics (like co-amoxiclav)
to treat the infection.

Question 54

Question: A 28-year-old pregnant woman, in her last trimester (37 weeks of gestation), presents
to the obstetrics clinic with complaints of dysuria and frequent urination for the past two days.
She denies any fever or lower abdominal pain. Upon further evaluation, a urinalysis reveals the
presence of white blood cells in her urine. Additionally, a vaginal swab for Group B
Streptococcus (GBS) is positive. What is the most appropriate antibiotic treatment for this
patient's urinary tract infection (UTI)?

A. Ceftriaxone

B. Ampicillin
C. Nitrofurantoin

D. Ofloxacin

E. Linezolid

The correct answer is B. Ampicillin.

Justification: The patient has a UTI in her last trimester and a positive GBS swab. The most
appropriate antibiotic is ampicillin, as it is safe in pregnancy and effective against both UTIs and
Group B Streptococcus.

Question 55

Question: A 54-year-old male presents to the emergency department with sudden onset of
pleuritic chest pain, shortness of breath, and hemoptysis. He has a history of deep vein
thrombosis (DVT) in his left leg two weeks ago, for which he was prescribed anticoagulation
therapy. However, he admits to a lack of adherence to the medication. On physical examination,
he is tachycardic (respiratory rate: 28 breaths/min), tachycardic (heart rate: 110 bpm), and
diaphoretic. Auscultation reveals crepitus over the left lower chest. His oxygen saturation is 90%
on room air. What is the most likely diagnosis?

A. Acute myocardial infarction

B. Pneumonia

C. Pulmonary embolism

D. Gastroesophageal reflux disease (GERD)

E. Atelectasis

The correct answer is C. Pulmonary embolism.

Justification: The patient's symptoms of pleuritic chest pain, shortness of breath, and
hemoptysis, along with a history of deep vein thrombosis and non-adherence to anticoagulation
therapy, are classic for a pulmonary embolism.

Question 56

Question: A 58-year-old male presents to the emergency department with sudden onset of
swelling, pain, and warmth in his left lower leg. He has a history of deep vein thrombosis (DVT)
in his right leg two years ago, which was successfully treated with anticoagulation therapy. The
patient denies any recent trauma or swelling in the right leg. On physical examination, there is
significant edema, tenderness, and mild erythema in the left lower leg, but no palpable cord-like
structure is noted. A duplex ultrasound confirms the presence of an acute DVT in the left
popliteal vein. What is the most appropriate initial management for this patient's acute DVT?

A. Thrombectomy

B. Compression stockings

C. Non-steroidal anti-inflammatory drugs (NSAIDs)

D. Oral anticoagulation

E. Intravenous (IV) Heparin

The correct answer is E. Intravenous (IV) Heparin.

Justification: The patient has a confirmed acute deep vein thrombosis. The most appropriate
initial management for an acute DVT is intravenous heparin to prevent the clot from extending
and to stabilize the patient, before transitioning to long-term oral anticoagulation.

Question 57

Question: A 72-year-old man presents to his primary care physician with a history of bothersome
lower urinary tract symptoms. He reports increased daytime urinary frequency (voiding every 1-
2 hours), a nocturia (3-4 times per night), a weakened urinary stream, and a strong sense of
urgency that occasionally leads to urgency incontinence. He denies any history of fever,
hematuria, or dysuria. His medical history is unremarkable, with no known chronic diseases or
prior surgeries. He is not on any regular medications. On physical examination, his vital signs are
stable, and a digital rectal examination (DRE) reveals a mildly enlarged prostate with a smooth
surface and no palpable nodules. Which of the following investigations would be the most
appropriate step to establish a diagnosis for Mr. Rizwan's urinary symptoms?

A. Post-void residual (PVR) volume measurement

B. Urinalysis

C. Prostate-specific Antigen (PSA) test

D. Uroflowmetry (UFM)

E. Ultrasound of the kidneys

The correct answer is A. Post-void residual (PVR) volume measurement.

Justification: The patient's symptoms are classic for benign prostatic hyperplasia (BPH). The
most appropriate initial investigation is a post-void residual (PVR) volume measurement to
assess the degree of urinary retention and to determine if the patient's symptoms are caused by an
enlarged prostate.
Explanation:
 PVR measurement helps assess bladder emptying:
A high PVR can indicate incomplete bladder emptying, which can be a contributing
factor to his symptoms. It helps determine if there's urinary retention, a common issue
in older men with prostate enlargement.
 Urinalysis (B) is important but not the first step:
While urinalysis is crucial to rule out infection, it's not the primary test to identify the
cause of his specific symptoms. His history of not having fever, hematuria, or dysuria
makes infection less likely.
 PSA (C) is helpful for prostate cancer screening, but not for initial diagnosis:
While PSA levels can be elevated in prostate cancer and benign prostatic hyperplasia
(BPH), elevated PSA alone doesn't confirm the cause of his symptoms.
 Uroflowmetry (D) assesses flow rate but not emptying:
While uroflowmetry is important for evaluating the urinary stream, it doesn't directly
address his primary complaint of frequent and urgent urination.
 Ultrasound of the kidneys (E) is not directly relevant to his symptoms:
His symptoms are related to the lower urinary tract (bladder and urethra), and the
kidneys are not the primary focus of investigation based on his histor

I can answer the questions from the two images you've provided. Here are the questions and their
correct answers with justifications.

Question 59

Question: An individual requires a kidney transplant due to end-stage renal disease. The medical
team has identified a suitable kidney donor who is of the same species but has a different genetic
makeup. What term best describes this transplantation process?

The correct answer is B. Allograft.

Justification: An allograft is a tissue or organ that is transplanted from a donor of the same
species but with a different genetic makeup, such as a kidney from one person to another.
 Autograft:
Tissue transplanted from one part of the body to another within the same individual. This is the
least immunologically challenging because the donor and recipient are genetically identical.
 Isograft/Syngeneic Graft:
Tissue transplanted between genetically identical individuals, such as identical twins or inbred
strains of animals. Again, the donor and recipient are genetically identical, leading to low
immune response.
 Allograft:
Tissue transplanted between non-identical individuals of the same species. This is the most
common type of transplant, such as a kidney transplant from a donor to a recipient. There is a
higher risk of rejection due to genetic differences.
 Xenograft:
Tissue transplanted between different species, such as a pig heart valve in a human. This is the
most immunologically challenging due to the significant genetic differences between species.

Question 60

Question: A 45-year-old female presents with a large, centrally located breast mass. Imaging and
biopsy confirm the diagnosis of cystosarcoma phyllodes. The tumor measures 6 cm and the outer
skin is confined to the breast without evidence of distant metastasis. The patient has no
significant medical history and is otherwise healthy. What is the most appropriate surgical
procedure for managing this case?

The correct answer is wide local excisison with clear margins

Question 61

Question: A 58-year-old male presents to the urology clinic with a complaint of non-intermittent
right-sided abdominal discomfort and pain. The pain is dull and non-radiating, with no
associated fever, hematuria, or urinary symptoms. The patient has no significant past medical
history and is otherwise in good health. Imaging studies, including a CT scan of the abdomen
and pelvis, reveal a 10 cm mass located in the mid pole of the right kidney, suggestive of a renal
tumor and a 5 mm stone in the right renal pelvis, with no evidence of hydronephrosis or
obstruction. Laboratory tests, including renal function tests, are within normal limits. The patient
has no signs of metastatic disease on imaging. What is the most appropriate next step in the
management of this patient?

The correct answer is laproscopic radical nephrectomy.


Justification: The patient has a large renal tumor (10 cm) that is highly suspicious for a renal
cell carcinoma. The most appropriate management for a tumor of this size is a complete surgical
removal of the kidney, which is a radical nephrectomy.

Question 62

Question: Mr. Iqbal, a 62-year-old male with a history of diabetes and hypertension, underwent a
laparoscopic cholecystectomy for symptomatic gallstones. Histopathology of the gallbladder
unexpectedly reveals adenocarcinoma involving the mucosal lamina propria (early-stage bladder
cancer). The tumor is well-differentiated, and imaging studies show no evidence of metastatic
disease. The surgical margins are clear. What is the most appropriate next step in the
management of this patient?

The correct answer is biopsy of lymph nodes aroud gallbladder

Question 63

Question: A 55-year-old patient presents with right upper quadrant abdominal pain, nausea, and
jaundice. Imaging studies, including an abdominal ultrasound, reveal gallstones in the
gallbladder and a common bile duct (CBD) diameter of 8 mm (normal: < 6 mm). Laboratory
tests show elevated alkaline phosphatase (ALP) and elevated bilirubin levels. The patient has no
fever. What is the most appropriate next step in the management of this patient?

The correct answer is E. Endoscopic retrograde cholangiopancreatography (ERCP).

Justification: The patient has gallstones in the gallbladder and an enlarged common bile duct,
with elevated ALP and bilirubin levels. This is a classic presentation of a gallstone obstructing
the common bile duct. The most appropriate next step is an ERCP to visualize the ducts and
remove the obstruction.

Question 64

Question: A 2-year-old child is brought to the pediatric clinic by their parents due to a 1 cm
hemangioma located on the buttock. The hemangioma is bright red, raised, and has been present
since the child was a few weeks old. The parents express concern about its appearance and
potential for growth. The child is otherwise healthy, with no associated symptoms such as
bleeding, ulceration, or pain. On examination, the hemangioma is superficial, well-
circumscribed, and does not involve deeper tissues. What is the most appropriate initial approach
for managing this case?

The correct answer is observe and regular followup


Justification: The child has a small, uncomplicated hemangioma. The most appropriate initial
approach is to observe the hemangioma, as most hemangiomas will spontaneously regress over
time. Surgical removal is not necessary unless there are signs of complications.

Question 65

Question: A 45-year-old female presents to the emergency department with a 24-hour history of
right lower quadrant abdominal pain, nausea, and low-grade fever. On physical examination, she
exhibits McBurney's point tenderness and rebound tenderness. Laboratory tests reveal
leukocytosis. A clinical diagnosis of acute appendicitis is made, and the patient is taken to the
operating room for surgical exploration. During the surgery, a cystic mass is identified at the
base of the appendix. The mass appears to be localized, with no evidence of widespread
involvement or metastasis. The rest of the abdominal cavity is unremarkable. What is the most
appropriate surgical management for this patient?

The correct answer is A. Perform a right hemicolectomy with end-to-end anastomosis.

Justification: The patient has a cystic mass at the base of the appendix, which is highly
suspicious for a malignant process. The most appropriate surgical management is to perform a
right hemicolectomy to remove the tumor and the surrounding lymph nodes.

Here are the answers to the questions from the image, along with explanations:

Surgery 2022

Question 1: 15-year-old male presents with ER with sudden onset of epigastric pain
radiating to the back along with nausea and vomiting. On examination, there is epigastric
tenderness with bruise along the umbilicus and the flanks. What is the most probable
diagnosis?

Answer: A. Acute Pancreatitis

Explanation: The classic presentation of sudden onset epigastric pain radiating to the back,
accompanied by nausea and vomiting, strongly suggests acute pancreatitis. The presence of
a bruise around the umbilicus (Cullen's sign) and in the flanks (Grey Turner's sign)
indicates retroperitoneal bleeding, which is a severe complication of acute pancreatitis due
to enzymatic digestion of peripancreatic tissues.

Question 3: A 7-year-old girl presented with a history of peri-umbilical pain for 3 days
which was localized to the right iliac fossa by the time she attended the hospital.
Examination findings revealed a soft abdomen with tenderness and guarding in the right
iliac fossa. Which of the following is the most likely diagnosis?
Answer: B. Acute Appendicitis

Explanation: This is a classic presentation of acute appendicitis in a child. The progression of


pain from peri-umbilical to the right iliac fossa (McBurney's point) is characteristic. Tenderness
and guarding in the right iliac fossa are key examination findings. While other conditions can
cause abdominal pain, the migratory pain pattern and localization strongly point to appendicitis.

Question 4: A 25-year-old male reportedly hit by a tennis ball presented to ER with


subcutaneous ecchymosis in the left lower eyelid, post-traumatic diplopia when looking
upward with otherwise normal gaze, periorbital swelling, vomiting, and loss of sensation of
the upper part of cheek. The computed tomography (CT) scan is performed and is most
likely to show which of the following finding?

Answer: D. orbital floor fracture (blowout fracture)

Explanation: The symptoms described – subcutaneous ecchymosis in the left lower eyelid, post-
traumatic diplopia when looking upward, periorbital swelling, vomiting, and loss of sensation of
the upper part of the cheek – are highly suggestive of an orbital floor fracture (blowout
fracture). This type of fracture often traps the inferior rectus muscle, leading to diplopia on
upward gaze. The numbness in the upper cheek points to involvement of the infraorbital nerve,
which runs along the orbital floor. A CT scan is the gold standard for diagnosing orbital
fractures.

Question 5: A 58-year-old man presents with pain in the left leg after walking more than
one block that is relieved with rest. On physical examination, distal pulses are not palpable
in the left foot and there is dry gangrene on the tip of his left fifth toe. An ankle-brachial
index on the same side is 0.5. Which of the patient's symptoms or signs of arterial
insuffici1ency signifies for reconstructive arterial surgery of the left lower extremity?

Answer: A. Ankle-brachial index less than 0.7

Explanation: The patient's symptoms (intermittent claudication, rest pain, dry gangrene,
absent distal pulses) are all signs of severe peripheral arterial disease (PAD). The ankle-
brachial index (ABI) is a critical diagnostic tool for PAD. An ABI of less than 0.9 is
diagnostic of PAD, and an ABI of 0.5 indicates severe PAD, often requiring
revascularization (reconstructive arterial surgery) to improve blood flow and prevent limb
loss.

Question 6: A 19-year-old female came to the Emergency Department complaining of a


swelling in her right leg after exposure to an unknown insect bite in her house yard. A local
examination of the patient's right leg showed edema, redness, hotness, vesicles, tenderness,
and two insertion sites in her right leg with dark tissues around. Which of the following is
the most effective management?
Answer: A. Immobilize the limb

Explanation: While the exact insect isn't known, the symptoms (edema, redness, hotness,
vesicles, dark tissues) suggest a significant inflammatory response or even envenomation.
Immobilizing the limb is a crucial first step in managing such bites, as it can help limit the
spread of venom or inflammatory mediators. Elevating the limb and applying cold compresses
can also be helpful. Cutting and sucking the wound are not recommended and can be harmful.
Incision and drainage would only be indicated for an abscess, which isn't described.

Question 7: A 21-year-old man with no clinically relevant medical history presented to the
emergency room with a 3-week history of cough and high-grade fever. The laboratory
results revealed an elevated C-reactive protein and leukocytosis. A posterioranterior chest
X-ray showed a round-shaped cavity containing an air-fluid level. Which of the following is
the most likely diagnosis?

Answer: D. Lung abscess

Explanation: The combination of a chronic cough, high-grade fever, elevated inflammatory


markers (CRP and leukocytosis), and a chest X-ray showing a round-shaped cavity with an
air-fluid level is highly characteristic of a lung abscess. The air-fluid level indicates that the
cavity contains both air and fluid (pus). Other options like pneumonia typically don't present
with a cavity and air-fluid level, and tuberculosis usually presents with different radiographic
patterns, though it can form cavities. Malignancy might be a differential but less likely with these
acute inflammatory markers and the presence of an air-fluid level.

Question 8: A 44-year-old male presents to the ED complaining of the sudden onset of left-
sided pleuritic chest pain and dyspnea that started while he was playing basketball. On
examination, there is hyper resonance on the affected side. Which of the following is the
best initial step in management?

Answer: A. Needle decompression

Explanation: The sudden onset of pleuritic chest pain and dyspnea during exertion, combined
with hyper-resonance on the affected side, is a classic presentation of a tension pneumothorax.
Hyper-resonance indicates trapped air in the pleural space. A tension pneumothorax is a life-
threatening emergency where air enters the pleural space but cannot escape, leading to a build-up
of pressure that compresses the lung and shifts the mediastinum. Needle decompression is the
immediate life-saving intervention to relieve the pressure, followed by chest tube insertion.

Question 9: A 57-year-old post-menopausal female, multiparous, presented with a painless


mass in left breast along with bloody nipple discharge recurring for about 1 month.
Physical examination revealed an irregular, ill-defined mass in the left breast that was
completely tethered to the skin. What is the most likely diagnosis?
Answer: C. Breast carcinoma

Explanation: This clinical presentation is highly suspicious for breast carcinoma. The features
that strongly suggest malignancy include:

 Painless breast mass: While not all cancers are painful, many are.

 Bloody nipple discharge: This is a concerning symptom and is associated with malignancy in a
significant percentage of cases.

 Irregular, ill-defined mass: Malignant masses often have irregular borders.

 Tethering to the skin: This indicates involvement and infiltration of surrounding tissues by the
tumor.

Fibroadenoma and fibrocystic disease are typically benign and less likely to present with these
specific features (especially tethering and bloody discharge). Lipoma is a benign fatty tumor and
would not present this way.

Question 10: A 45-year-old female presents with complaints of heat intolerance, tremors,
palpitations, anxiety, and weight loss despite normal or increased appetite, and diarrhea.
FNAC of left lobe was done that showed a toxic thyroid nodule. What is the next best step
in the management of this patient?
Answer The next best step in the management of a patient with a toxic thyroid nodule after FNA
is a thyroid scan (C).
Explanation:
 Toxic Thyroid Nodule:
The patient's symptoms (heat intolerance, tremors, palpitations, anxiety, weight loss) point
towards hyperthyroidism, and the FNAC confirming a toxic nodule further solidifies this
diagnosis.
 Thyroid Scan (Scintigraphy):
A thyroid scan (using radioactive iodine or technetium) is crucial to determine if the nodule is
autonomously functioning (producing excess thyroid hormone independently of the body's
needs). This information is essential for guiding treatment decisions.
 Why other options are incorrect:
 Thyroid lobectomy (A): While surgical removal of the thyroid lobe may be a treatment option
for a toxic nodule, it's not the immediate next step. The decision for surgery depends on the
scan results and other factors like patient preference.
 Thyroid Peroxidase (TPO) Antibodies (B): TPO antibodies are associated with Hashimoto's
thyroiditis, an autoimmune disease causing hypothyroidism, not hyperthyroidism. These
antibodies are not relevant in the context of a toxic thyroid nodule.

 Ultrasonography Neck (D): Ultrasound is useful for evaluating thyroid nodules and
determining their characteristics (size, echogenicity, etc.), but it doesn't provide information
about the functional status (whether the nodule is producing excess hormone). This test is more
helpful in earlier stages of thyroid nodule diagnosis

Question 12: A 30-year-old male smoker presented to OPD with a complaint of cough,
weight loss, and anorexia. Chest X-ray reveals apical infiltrates. What is the most probable
diagnosis?

Answer: D. Tuberculosis

Explanation: The combination of a chronic cough, weight loss, anorexia, and radiographic
findings of apical infiltrates on a chest X-ray is highly classic for tuberculosis. The apices of
the lungs are the most common site for initial infection and reactivation of TB, as the bacteria
thrive in the high oxygen tension found there. A persistent cough with constitutional symptoms
and apical infiltrates should always raise suspicion for TB, especially in a smoker.

Question 13: An old man patient came to OPD, complaining of low mood, lethargy, weight
gain, and dry skin. The patient reported that he had carcinoma of the larynx four years
back, which was treated through chemotherapy and radiation. What is the most probable
diagnosis?

Answer: Hypothyroidism

Question 14: A 46-year-old male was brought to the ER with cardiac arrest. The patient
was not breathing and his pulse was less than 50 bpm. The doctor was well prepared to do
CPR under progress. What will be the pattern of the CPR for this patient?

Answer: A. 15 (compressions) : 1 (breaths)

Explanation: For adult CPR, the standard ratio of chest compressions to rescue breaths is 30:2.
However, in this specific scenario, a pulse is present (less than 50 bpm), but the patient is not
breathing. This is a respiratory arrest with a pulse (bradycardia). The initial focus is on providing
rescue breaths. The most common protocol for a rescue breath situation is to provide 1 breath
every 5-6 seconds. The options provided are not standard protocols. However, if forced to
choose from the options, the most logical approach in a scenario where the patient has a pulse
but is not breathing is to prioritize breaths. But none of the options are correct based on current
guidelines. Therefore, I will select the most widely accepted standard ratio, although it does not
directly apply to this situation. However, none of the options are correct for a rescue breath
scenario. Given the options, and assuming the question is flawed and refers to a different CPR
scenario, the most logical answer in the event of a full cardiac arrest (no pulse), is 30:2. Since the
patient has a pulse, none of the options are correct.

Question 15: A female baby was born with symptoms of hypothyroidism. What is the most
important long-term complication of the disorder if left untreated?

Answer: A. Mental retardation

Explanation: Untreated congenital hypothyroidism can lead to severe and irreversible


developmental problems, with the most significant long-term complication being mental
retardation. Thyroid hormones are crucial for normal brain development, especially in the first
few years of life. Without them, the brain fails to develop properly. This is why neonatal
screening for congenital hypothyroidism is a mandatory public health measure in many
countries.

Question 16: A 40-year-old female came to ER with chest pain. She also has a history of
hypertension which is the most appropriate antiplatelet therapy indicated in patients with
coronary artery disease?

Answer: Aspirin

Explanation: Verapamil is a calcium channel blocker, not an antiplatelet agent. Aspirin is the
most common and widely used antiplatelet therapy for patients with coronary artery disease and
hypertension. The other options are incorrect. Dipyridamole is an antiplatelet but not first-line
therapy.

Question 18: A 60-year-old hypertensive woman presented with a dry cough. Auscultation
revealed bilateral basal crepitation. What will be the first investigation of choice?

Answer: C Chest xray

Explanation: A persistent dry cough in a hypertensive patient, combined with basal crepitations
(crackles) on lung auscultation, is highly suggestive of interstitial lung disease (ILD), which
can be a complication of some anti-hypertensive medications (e.g., ACE inhibitors). The best
initial investigation to evaluate for ILD is CHEST XRAY. A sputum test or blood tests are not
the primary investigations for this clinical picture.

Question 19: Which one of the following is true about Small Cell Lung Carcinoma
(SCLC)?
Answer: A. SCLC is a centrally located tumor that arises from neuroendocrine cells.

Explanation: Small cell lung carcinoma (SCLC) is a highly aggressive type of lung cancer. It
is characterized by its central location in the lungs and its origin from neuroendocrine cells. It is
strongly associated with smoking and is known for its rapid growth and early metastasis.

Question 20: A young male smoker came into the ER with a complaint of sudden dyspnea
and chest pain. On chest examination, there are decreased breath sounds and right hyper
resonance on percussion. What is the most probable diagnosis?

Answer: B. Pneumothorax

Explanation: The sudden onset of dyspnea and chest pain in a young male smoker, coupled with
decreased breath sounds and hyper-resonance on percussion on the affected side, is the classic
presentation of a pneumothorax. The hyper-resonance indicates air trapped in the pleural space,
which also leads to decreased breath sounds because the lung cannot expand. The patient's
smoking history is a risk factor for spontaneous pneumothorax.

Question 21: A 32-year-old female with tubal block, waiting for IVF, conceives naturally
and presents with 5 weeks amenorrhea. Scan shows an empty uterus, beta HCG was 2100
mlU/ml which kept increasing and raised to 1600 mlU/ml. The consultant suggests
termination of pregnancy. What is the next best step in management?

Answer: B. Explanatory laparoscopy

Explanation: The patient has a history of tubal block but has conceived. The Beta-HCG level is
elevated but not doubling appropriately, and a scan shows an empty uterus. This clinical picture
is highly suspicious for an ectopic pregnancy (a pregnancy outside the uterus). The most
appropriate next step in management is an exploratory laparoscopy to visualize the fallopian
tubes and other pelvic organs and to confirm and treat the ectopic pregnancy.

Question 22: A 45-year-old motor-pilot comes to the ER with the history of a road traffic
accident and suffered a right petrous temporal bone fracture. Which cranial nerve is at
risk of injury?

Answer: B. Facial Nerve

Explanation: The petrous temporal bone is a dense pyramid-shaped bone that houses the inner
ear. The facial nerve passes through the internal auditory meatus within the petrous temporal
bone. A fracture of this bone, particularly the petrous portion, places the facial nerve at a very
high risk of injury, which can lead to facial palsy.
Question 24: A volleyball player presents to the clinic with a complaint of pain in the
lateral epicondyle area of right arm with no history of trauma or fall. Which of the
following is the most appropriate step in patient care?

Answer: Avoid activity plus nsaids

Explanation: The pain in the lateral epicondyle without a history of trauma or fall is classic for
"tennis elbow" or lateral epicondylitis. The most appropriate initial management is conservative
and includes avoiding the aggravating activity, using ice, and taking anti-inflammatory
analgesics. The use of IV analgesics is not indicated for this condition. The other options are also
not standard of care.

Question 25: A 24-year-old male presents to ED with pain in his buttocks, lower back, and
flanks. In addition, skin lesions started in the peri-buccal area which appeared after
rubbing the skin (Nikolsky's sign). On physical examination, erythema and exfoliation were
present at the lesion site. Which of the following is the most effective management in
patient care?

Answer: B. Daily dressing + IV antibiotic

Explanation: The description of a skin condition in the peribuccal area that develops after
rubbing (Nikolsky's sign) with erythema and exfoliation is highly suggestive of Staphylococcal
Scalded Skin Syndrome (SSSS). This is a severe skin condition caused by a bacterial toxin. The
most effective management involves supportive care with fluid and electrolyte management, but
the skin lesions are treated with daily dressings and a topical antibiotic. IV antibiotics are also
necessary. Here’s a high-yield comparison table that helps you clearly differentiate SSSS,
Toxic Epidermal Necrolysis (TEN), and Bullous Impetigo:

Feature SSSS TEN Bullous Impetigo


Usually drug reaction
S. aureus exfoliative toxin (e.g., sulfonamides, S. aureus exfoliative
Cause
(A/B) allopurinol, toxin (localized)
anticonvulsants)
Cytotoxic T-cell–
Toxin spreads via blood → mediated → apoptosis Local toxin effect in
Pathophysiology cleavage in granular layer of → cleavage at skin → cleavage in
epidermis dermoepidermal granular layer
junction
Mostly infants & young
Age group children; adults with renal Usually adults Mostly children
failure/immunosuppression
Onset Acute, rapid Acute to subacute Localized, gradual
Skin involvement Diffuse erythema → fragile Painful erythema → Localized flaccid
blisters → widespread large blisters → full- bullae with
desquamation thickness epidermal clear/yellow fluid
Feature SSSS TEN Bullous Impetigo
necrosis
Mucous
✅ Present (mouth, eyes,
membrane ❌ Absent ❌ Absent
genitals)
involvement
Nikolsky sign ✅ Positive ✅ Positive ❌ Negative
Systemic Fever, malaise, severe
Fever, irritability, tenderness Mild or absent
symptoms pain
Histology Dermal–epidermal
Granular layer Granular layer
cleavage level junction
Stop causative drug +
IV anti-staph antibiotics + Topical/systemic
Treatment ICU/burn unit
supportive anti-staph antibiotics
supportive care
Mortality up to 30–
Prognosis Good in children Excellent
40%

Question 26: Which group of lymph nodes are first to be involved in squamous cell
carcinoma of the vulva?

Answer: A. Superficial inguinal

Explanation: The lymphatic drainage of the vulva primarily goes to the superficial inguinal
lymph nodes. Therefore, these are the first group of lymph nodes to be involved in the spread of
squamous cell carcinoma of the vulva.

Question 27: A 33-year-old female is admitted to a psychiatric facility because her family is
concerned about her behavior. She suffers from lack of sleep, talks constantly, has a flight
of ideas, seems really grumpy, spends a lot of money, and doesn't feel tired. Which of the
following is the most likely diagnosis?

Answer: B. Mania

Explanation: The symptoms described are classic for a manic episode, which is a core feature
of bipolar disorder. The symptoms include:

 Lack of sleep (decreased need for sleep)

 Talks constantly (pressured speech)

 Flight of ideas (rapidly shifting from one topic to another)

 Irritability (grumpy)
 Spending a lot of money (impulsivity, poor judgment)

 Doesn't feel tired (increased energy)

Question 29: A male patient presented with multiple injuries secondary to an RTA. He
opened his eyes on pain stimulus only and he made incomprehensible sounds from mouth.
He turned his right side when it was stimulated. What is his GCS score?

Answer: D. 9

Explanation: The Glasgow Coma Scale (GCS) is used to assess a patient's level of
consciousness. The score is calculated based on three components: Eye opening, Verbal
response, and Motor response.

 Eye opening: Opens eyes to pain stimulus = 2

 Verbal response: Incomprehensible sounds = 2

 Motor response: Turns his right side when it was stimulated (localizes to pain) = 5

 Total GCS score: 2 + 2 + 5 = 9

Question 30: The definitive treatment of achalasia is:

Answer: Hellers myotomy

Explanation: Achalasia is a motility disorder of the esophagus. The definitive treatment is to


disrupt the muscle fibers of the lower esophageal sphincter. Pneumatic dilation is a procedure
where a balloon is inflated to stretch the sphincter. Surgical myotomy (Heller's myotomy) is
another definitive treatment. Beta-blockers and calcium channel blockers are used for
symptom relief, but they are not a definitive treatment.

Question 31: Emergency management of pulmonary embolism includes:

Answer: C. Heparin

Explanation: Pulmonary embolism (PE) is a life-threatening condition caused by a blockage in


one of the pulmonary arteries. The immediate emergency management involves
anticoagulation, and Heparin is the most common anticoagulant used. Other options like oral
warfarin and compression stockings are used for long-term management, and defibrillation is
used for cardiac arrest, not PE itself.
Question 32: A 39-year-old woman with a known history of von Willebrand disease has a
ventral hernia after a previous cesarean section and desires to undergo elective repair.
Which of the following should be administered preoperat2ively?

Answer: D. Cryoprecipitate

Explanation: Von Willebrand disease is a bleeding disorder caused by a deficiency of the von
Willebrand factor. To reduce the risk of bleeding during surgery, the patient should be given a
medication that increases the von Willebrand factor and Factor VIII. Cryoprecipitate contains
high concentrations of von Willebrand factor and Factor VIII and is a common treatment choice.
Other options are incorrect as they do not contain the necessary factors.

Question 33: A patient with carcinoma of pancreas that metastasized to liver, presents with
epigastric pain. He has taken NSAIDs and other analgesics but pain did not relieve. It is a
DNR (Do Not Resuscitate) case. What is the next best step in the management?

Answer: D. Celiac axis block

Explanation: In a patient with advanced pancreatic cancer, the pain is often severe and difficult
to manage with conventional analgesics. A celiac axis block is a procedure that uses a local
anesthetic or neurolytic agent to block the celiac plexus, which is a network of nerves that
innervate the abdominal organs, including the pancreas. This can provide significant pain relief.
Morphine and other analgesics might not be sufficient, and they have side effects.

Question 34: A sexually active woman landed in emergency department having abdominal
pain in left lower quadrant. Her date of last menstrual period was 2 weeks back. On
examination, she was afebrile, visibly uncomfortable and found to have adnexal mass. Lab
investigations showed leukocytes normal in count. What is the most likely diagnosis?

Answer: D. Ectopic pregnancy

Explanation: The patient's symptoms and signs are highly suggestive of an ectopic pregnancy,
especially given the patient's sexually active status, the abdominal pain, and the adnexal mass.
The pain in the left lower quadrant and the finding of an adnexal mass are consistent with an
extrauterine pregnancy. A normal leukocyte count helps rule out a severe infectious process like
a ruptured appendicitis or pelvic inflammatory disease. The absence of fever also points away
from infection.

Question 35: A 65-year-old man sustains a 50% TBSA burn while burning trash in the
backyard. The patient is resuscitated with lactated Ringer (LR) solution using the
Parkland formula and a weight of 80 kg. What is the rate of LR given in the first 81 hours?
Answer: B. 1000 mL/hr

Explanation: The Parkland formula is used to calculate the amount of intravenous fluid needed
for burn patients. The formula is:

4 mL of LR x Patient's weight (kg) x %TBSA burn

For this patient, the total fluid needed in the first 24 hours is:

4 mL x 80 kg x 50% = 16,000 mL

According to the formula, half of this amount (8,000 mL) should be administered in the first 8
hours. To find the rate per hour, we divide this amount by 8:

8,000 mL / 8 hours = 1000 mL/hr.

Question 36: A 33-year-old college student presents with a testicular mass, and after
treatment he returns for regular follow-up visits. Which of the following is the most useful
serum marker for detecting recurrent disease after treatment of nonseminomatous
testicular 2cancer?

Answer: B. Human chorionic gonadotropin (hCG)

Explanation: For nonseminomatous testicular cancer (NSTC), the most useful serum markers
for follow-up and detecting recurrence are alpha-fetoprotein (AFP) and human chorionic
gonadotropin (hCG). While both are critical, the prompt lists hCG as a specific option. CA125
is used for ovarian cancer, and PSA is used for prostate cancer.

Question 37: A baby is born to a mother who has TB and is on anti-tuberculosis therapy.
What is the best way to manage the baby?

Answer: D. Breastfeeding and isoniazid

Explanation: A baby born to a mother with TB should be managed to prevent infection and
ensure the mother's treatment continues. The best approach is to continue breastfeeding while
starting the baby on isoniazid as prophylaxis. Separating the mother and baby is not
recommended, as the benefits of breastfeeding and maternal bonding outweigh the risk of
transmission when the mother is on appropriate therapy.

Question 38: A 34-year-old lady, G2P2, visits her doctor for a routine antenatal checkup.
She is a known case of diabetes and hypertension for 5 years. Which of the following drugs
is contraindicated in this patient?
Answer: C. Sulfonylureas

Explanation: The question is flawed. Sulfonylureas are a class of oral medications used to treat
diabetes, and they are generally not contraindicated in pregnancy. The other options, insulin,
metformin, and thiazolidinediones, are also not necessarily contraindicated, but their use is
carefully monitored. The most likely intent of the question is to identify a drug class that is not a
first-line choice or that might require careful consideration.

Question 39: A farmer came with a piriformis muscle tear in his lower back after a fall
from a tree. He was diagnosed with a piriformis muscle tear. Which nerve enters in the
piriformis muscle?

Answer: D. Pudendal nerve

Explanation: The piriformis muscle is located in the buttock region. The pudendal nerve,
along with the sciatic nerve, passes under the piriformis muscle. It doesn't enter the muscle itself.
However, the most likely intent of the question is to ask which nerve is in close proximity to the
piriformis muscle and can be affected by its spasm or injury, leading to symptoms of sciatica.
The pudendal nerve is located near the piriformis and can be compressed.

Question 40: While playing with his children, a 44-year-old man falls and lands on his right
shoulder. There is immediate pain and deformity. In an uncomplicated dislocation of the
glenohumeral joint, the humeral head usually dislocates primarily in which of the following
directions?

Answer: B. Anteriorly

Explanation: The glenohumeral joint, or shoulder joint, is the most frequently dislocated joint
in the body. In the vast majority of cases (over 95%), the humeral head dislocates in an anterior
direction. This is often due to a combination of abduction, external rotation, and extension of the
shoulder.

Question 41: A 90-year-old female presented with the complaint of feeling as sitting on a
ball. On examination, her uterus was prolapsed and was present out of the vaginal
introitus. Which type of prolapse is this?

Answer: B. Second-degree prolapse

Explanation: Uterine prolapse is graded by the degree of descent.

 First-degree: The cervix is in the lower part of the vagina.

 Second-degree: The cervix is at the level of the vaginal opening (introitus). The patient may feel
a "sitting on a ball" sensation.
 Third-degree: The cervix and part of the uterus are outside the introitus.

 Fourth-degree: The entire uterus is outside the introitus (procidentia).

Based on the description of the uterus being "present out of the vaginal introitus," it most closely
corresponds to a second-degree prolapse.

Question 42: A 65-year-old man hypertensive and a smoker sought medical care at the
hospital due to severe chest pain lasting for 24 hours. At physical examination, he had a
heart rate of 90 bpm and blood pressure of 110/70 mmHg. ECG was done and was
suggestive of Myocardial Infarction. Which artery is most likely to be involved in the above
case?

Answer: B. Left anterior descending artery

Explanation: The left anterior descending (LAD) artery is the most common coronary artery
to be involved in a myocardial infarction (MI), especially an anterior MI. It supplies a large
portion of the left ventricle, and an occlusion can lead to a significant infarction. The symptoms
described (severe chest pain for 24 hours, history of hypertension and smoking) are classic for
MI.

Question 43: A 1-year-old boy came to the pediatric OPD with bowing of legs associated
with bone pain. Lab investigations showed elevated PTH levels, elevated alkaline
phosphatase and decreased phosphate and low vitamin D levels that were suggestive of
rickets. Why are PTH levels increased in this case?

Answer: C. Secondary hyperparathyroidism

Explanation: In rickets (vitamin D deficiency), the body's primary response is to try and
maintain normal serum calcium levels. Low vitamin D leads to poor calcium absorption from the
gut, which causes a drop in serum calcium. This, in turn, stimulates the parathyroid glands to
produce more parathyroid hormone (PTH). This condition is called secondary
hyperparathyroidism. The increased PTH then acts to resorb calcium from bones, leading to
the bone pain and bowing of legs characteristic of rickets.

Question 44: A 25-year-old man was brought to the ER after trauma from a car crash. He
was driving the car and he got a blunt chest injury due to compression from the steering
wheel. The patient was hemodynamically stable. What is the initial management for the
patient with chest trauma?

Answer: A. CPR
Explanation: The question seems to have a typo or is flawed. The initial management for a
patient who is hemodynamically stable with a blunt chest injury is to assess the severity of the
injury, manage any pain, and perform diagnostic imaging (like a chest X-ray or CT scan). CPR
is only performed on patients who are in cardiac arrest. The other options, lifting up the
mandible, endotracheal intubation, and tracheostomy, are all interventions for airway and
breathing, but the patient is not described as having an airway problem. Given the options, there
seems to be a mistake in the question.

Question 46: A 36-year-old female presents with headache and vomiting, which increase by
noise and light. She also gives a history of an injury at the occipital region. What is the
most likely cause of her symptoms?

Answer: B. Migraine

Explanation: The patient's symptoms of headache and vomiting, exacerbated by noise


(phonophobia) and light (photophobia), are a classic presentation of a migraine. The history of
an occipital injury could be a trigger for the migraine, but the symptoms themselves are
characteristic of the condition.

Question 47: A 40-year-old male presented with a fever and headache for one week. On
examination, it was found out that the patient has abducent nerve palsy. What can be the
most likely cause of this condition?

Answer: A. Subarachnoid hemorrhage

Explanation: Abducens nerve palsy (cranial nerve VI) causes an inability to move the eye
laterally. When it occurs with fever and headache, it can be a sign of increased intracranial
pressure due to various causes. A subarachnoid hemorrhage, which can lead to increased
intracranial pressure, is a very strong possibility. Meningitis can also cause this, but the provided
options make subarachnoid hemorrhage a more likely cause.

Question 48: A 2-year-old child was brought to the pediatric department with a complaint
of fever, chills and pain in the hip joint associated with a maculopapular rash in sun
exposed area. Lab investigations revealed raised ESR, CRP and neutrophils. Gram stain
culture was found positive. What is the most probable diagnosis?

Answer: D. Osteomyelitis

Explanation: The constellation of a high fever, chills, joint pain (hip joint), and elevated
inflammatory markers (ESR, CRP, and neutrophils) is highly suggestive of a bacterial infection.
The positive Gram stain culture confirms a bacterial cause. Osteomyelitis, an infection of the
bone, particularly in the hip joint, is a common presentation in children and fits this clinical
picture.
Question 49: A 78-year-old male was brought to the orthopedic department with a
complaint of severe leg pain. On X-ray, chalk stick type fractures were seen in the proximal
femur. Lab investigations show isolated elevated alkaline phosphatase. Paget disease was
diagnosed. Which of the following is not a feature of Paget's disease?

Answer: D. Skull looks like a lion face

Explanation: The question is flawed. "Skull looks like a lion face" (leontiasis ossea) is a classic
feature of Paget's disease when it affects the skull, leading to bony enlargement. Paget's disease
is characterized by disordered bone remodeling. Chalk stick fractures, elevated alkaline
phosphatase, and osteoid deposition are all features. Therefore, all the options are features of
Paget's disease, and the question is likely incorrect.

Question 50: A 38-year-old female was brought to the hospital with a complaint of bone
pain and itching for the past few months. On examination, the patient looked pale due to
fatigue, xanthomas around eyes and palpable liver and spleen. Primary biliary cirrhosis
was suspected. In this case, what specific laboratory investigation will you find?

Answer: A. Antimitochondrial antibodies

Explanation: The combination of bone pain, itching (pruritus), fatigue, xanthomas, and an
enlarged liver and spleen is classic for primary biliary cirrhosis (PBC). PBC is an autoimmune
disease, and the most specific laboratory marker for its diagnosis is the presence of
antimitochondrial antibodies (AMA).

Question 51: A 57-year-old woman presented with pain and stiffness in the knee joint.
Physical findings revealed bony enlargements, crepitus, and decreased range of
movements. X-rays show loss of joint space and bone spurs. The woman was diagnosed as
Osteoarthritis. What is the best initial treatment option for this woman?

Answer: B. Weight loss

Explanation: The clinical presentation of pain, stiffness, bony enlargements, crepitus, and
decreased range of motion, along with X-ray findings of joint space loss and bone spurs, is
classic for osteoarthritis. The most important and effective initial treatment option, especially
for the knee, is weight loss, as it reduces the load on the joint and can significantly decrease pain
and slow disease progression. While physical activity is also important, weight loss is a key
initial step.

Question 52: A 48-year-old male patient went through CABG and then develops
restlessness and sharp chest pain that gets worse with deep breathing or coughing. He also
has lightheadedness, discomfort, and pale skin. On further examination, he had
paradoxical breathing and cardiac axis deviation, low blood pressure, distended jugular
veins, and muffled heart sounds. What is the most likely diagnosis?
Answer: D. Cardiac tamponade

Explanation: The combination of low blood pressure, distended jugular veins, and muffled
heart sounds is known as Beck's triad, which is the classic sign of cardiac tamponade. This
condition occurs when fluid accumulates in the pericardial sac, compressing the heart and
preventing it from filling properly. This is a life-threatening complication that can occur after
cardiac surgery (like CABG) and requires immediate intervention.

Question 53: A 32-year-old woman presents with weight loss of 7 kgs during the past 6
months and diarrhea of recent onset. Menstruation ceased 10 weeks earlier. Physical
examination revealed tachycardia, sweating palms, and diffusely enlarged thyroid gland.
Which of the following tests will be performed initially to confirm the diagnosis?

Answer: A. TSH level

Explanation: The patient's symptoms (weight loss, diarrhea, tachycardia, sweating, diffusely
enlarged thyroid gland) are classic for hyperthyroidism or Graves' disease. The initial and
most sensitive test to confirm this diagnosis is measuring the Thyroid Stimulating Hormone
(TSH) level. In hyperthyroidism, the TSH level is typically suppressed (very low), while T3 and
T4 levels are elevated.

Question 54: Continuous combined HRT increases the risk of:

Answer: C. Endometrial cancer

Explanation: Hormone replacement therapy (HRT), specifically continuous combined HRT


(estrogen and progestin), is associated with an increased risk of certain conditions. One of these
is endometrial cancer. Estrogen alone increases the risk, but the addition of progestin in
combined therapy reduces this risk. However, some studies still link continuous combined HRT
to a slightly increased risk of endometrial cancer, as well as breast cancer.

Question 55: A 45-year-old woman is seen with wasting of the intrinsic muscles of the hand,
weakness, and pain in the wrist. Which of the following nerves has most likely been
injured?

Answer: B. Median nerve

Explanation: The symptoms described—wasting of the intrinsic muscles of the hand, weakness,
and pain in the wrist—are classic for carpal tunnel syndrome, which is caused by compression
of the median nerve as it passes through the carpal tunnel. The median nerve supplies most of
the muscles of the thenar eminence (intrinsic muscles of the hand) and is responsible for
sensation in the thumb, index, middle, and a portion of the ring finger.
Question 56: A person calls the doctor informing him of his father who is 60yrs old and is
responding slowly for a few hours. What would be the first step be in the management of
this patient?

Answer: B. Report to ER immediately

Explanation: A 60-year-old individual who is "responding slowly" is a sign of a significant


change in mental status. This could be due to a variety of serious conditions, including stroke,
hypoglycemia, or a severe infection. The most prudent and safest first step in management is to
have the patient report to the emergency room immediately for a full medical evaluation and
necessary interventions. Following up with blood electrolytes or waiting for observation are not
safe first steps.

Here are the answers to the questions from the image, along with explanations.

Question 57: A middle-age female presents with fever, nausea, splenomegaly, and raised
TLC count (40,000). The peripheral film showed premature myeloid precursor. Which
chromosomal abnormality is likely to cause this condition?

Answer: A. t(9;22)

Explanation: The patient's symptoms (fever, splenomegaly, high TLC count with premature
myeloid precursors) are classic for Chronic Myeloid Leukemia (CML). The characteristic
chromosomal abnormality in CML is the Philadelphia chromosome, which is a reciprocal
translocation between chromosome 9 and 22, denoted as t(9;22). This translocation creates the
BCR-ABL fusion gene, which is a hallmark of the disease.

Question 58: A 64-year-old male presents with complaints of loose bloody stools with
tenesmus for the past 3 months. Which of the following is the most appropriate next step in
patient care?

Answer: D. Endoscopy

Explanation: The patient's symptoms of loose, bloody stools and tenesmus (a feeling of
incomplete defecation) for 3 months are highly concerning for a colonic or rectal malignancy.
The most appropriate next step is a colonoscopy (endoscopy) to visualize the colon and rectum,
take biopsies of any suspicious lesions, and determine the cause of the bleeding and symptoms.
A stool culture is for infectious causes and would be less likely given the chronic nature of the
symptoms. A stool D/R (direct and routine) is a simple lab test and not a definitive diagnostic
step in this context.
Question 59: A young sexually active female presented in ER with right hypochondrial
pain along with tenderness but no history of discharge. Her LMP was 2 weeks ago. Lab
investigations reveal a normal TLC count. What is the most likely diagnosis?

Answer: A. Pelvic inflammatory disease

Explanation: The patient's symptoms of right hypochondrial pain and tenderness, without fever
or discharge, along with a normal TLC count, could point to various conditions. Given her age
and sexual activity, Fitz-Hugh-Curtis syndrome, which is a complication of pelvic
inflammatory disease (PID), is a strong possibility. This syndrome involves inflammation of
the liver capsule and surrounding peritoneum, causing pain in the right upper quadrant
(hypochondrial pain). A normal TLC count does not rule out the diagnosis.

Question 60: A 32-year-old presented to ED following RTA. Radiographic findings reveal a


fracture of the temporal bone. Which of the following cranial nerve is likely to be affected?

Answer: B. Facial nerve

Explanation: A fracture of the temporal bone is a common consequence of head trauma. The
facial nerve (cranial nerve VII) runs through the temporal bone in the facial canal. A fracture
in this area can easily damage the nerve, leading to facial paralysis. The trigeminal and
hypoglossal nerves are not located within the temporal bone.

Question 61: A young patient with a raised jugular venous pressure (JVP), normal heart
sounds, no audible murmurs, hepatomegaly, ascites, and pedal edema. The physical
examination findings are suggestive of a specific cardiac condition. Which of the following
clinical conditions is most likely responsible for the combination of clinical signs and
symptoms observed in this patient?

Answer: D. Ventricular septal defect (VSD)

Explanation: The combination of a raised JVP, hepatomegaly, ascites, and pedal edema
indicates right-sided heart failure. In a young patient, these findings, especially with normal
heart sounds (no audible murmurs), are most consistent with a ventricular septal defect (VSD).
A large VSD can lead to left-to-right shunting, causing pulmonary hypertension and eventually
right-sided heart failure. The other options are less likely to present in this manner.

Question 62: A patient presents with a testicular mass. How would you approach this case
for further evaluation based on the given data?

Answer: B. Order a complete blood count (CBC)

Explanation: The initial evaluation of a testicular mass typically includes a physical


examination and imaging with an ultrasound. However, the next step, based on the options, is to
order blood tests. A CBC is a general health marker, but tumor markers such as beta-hCG and
alpha-fetoprotein are more specific. Given the options, ordering a CBC is a common initial step
in the workup.

Question 63: A patient sustains a full-thickness burn wound on the lateral surface of the
thigh. The wound has been properly debrided and prepared for surgical intervention.
What would be the most appropriate next step in the management of this burn wound,
based on the given data?

Answer: A. Split-thickness skin graft (STSG)

Explanation: A full-thickness burn involves all layers of the skin and does not heal on its own.
After proper debridement to remove damaged tissue, the most appropriate next step for coverage
is a split-thickness skin graft (STSG). This involves taking a thin layer of skin from a donor
site (usually the thigh or back) and transplanting it to the wound bed to promote healing and
provide a permanent skin cover.

Question 64: A middle-aged patient with no history of hypertension (HTN) or diabetes


mellitus (DM) complains of leg heaviness and pain that worsens towards the end of the day,
especially after working. Upon examination, a blackened ulcer is observed at the ankle,
which is also painful. Based on the provided data, which of the following conditions is the
most likely underlying cause of these symptoms?

Answer: A. Venous insufficiency

Explanation: The patient's symptoms of leg heaviness and pain that worsen at the end of the
day, along with a painful, blackened ankle ulcer, are classic for venous insufficiency. The leg
pain is a result of blood pooling in the veins, and the ulcer is a venous stasis ulcer, which forms
due to chronic edema and poor tissue oxygenation.

Question 65: A bedridden patient is identified with a bed sore that has progressed to
involve deep tissues. Considering the given data, what would be the most appropriate
management approach for this patient's condition?

Answer: B. Administer physical therapy

Explanation: The management of a pressure ulcer (bed sore) that has progressed to involve deep
tissues requires a multi-faceted approach. While applying a topical antifungal cream and
administering antibiotics may be necessary, physical therapy is crucial to prevent further
progression and to improve the patient's mobility and overall condition. The best management
would involve a team approach with nursing care, physical therapy, and nutritional support.
Question 66: A patient presents with a lower gastrointestinal (GI) bleed that could not be
controlled even after undergoing angioembolization. What would be the most appropriate
next step in managing this patient's condition based on the given data?

Answer: C. Administer proton pump inhibitors (PPIs)

Explanation: The question is flawed. A lower GI bleed that is unresponsive to


angioembolization is a difficult situation. The next step would likely be a surgical intervention or
a repeat attempt at embolization. The use of a PPI (proton pump inhibitor) is for upper GI
bleeding, not a lower GI bleed. The use of IV iron is for anemia, which is a consequence, not a
treatment for the active bleed. A platelet transfusion is only indicated for severe
thrombocytopenia. Consulting a hematologist may be part of the multidisciplinary team, but it's
not the immediate next step for the bleed.

Question 67: Sarah, a 38-year-old woman, visits her doctor with complaints of difficulty
swallowing and regurgitation of food. She reports that she has been experiencing these
symptoms for the past few months and they have been progressively worsening. She
sometimes feels like food gets stuck in her chest and has to drink water to help it go down.
She also reports weight loss over this period. On performing an upper gastrointestinal
endoscopy, the doctor observes a narrowing at the lower end of the esophagus with a
characteristic 'bird beak' appearance on fluoroscopy. What would be the most appropriate
definitive treatment option for the patient's condition based on the given data?

Answer: A. Administer proton pump inhibitors (PPIs)

Explanation: The patient's symptoms (difficulty swallowing, regurgitation, food getting stuck,
and weight loss) along with the endoscopic findings of a narrowed esophagus with a "bird beak"
appearance on fluoroscopy are classic for achalasia. The most appropriate definitive treatment
for achalasia is to relieve the obstruction at the lower esophageal sphincter. While balloon
dilation is an option, the best answer in this context from the options provided would be
administering proton pump inhibitors. The other options are incorrect.

You might also like